× Register Login What's New! Contact us
Page 2 of 7 First 1 2 3 4 ... Last
Results 21 to 40 of 124 visibility 495219

The Medical student Review

  1. #1
    brightness_1
    Soldier Through It!
    Full Member Array جوري's Avatar
    Join Date
    Jul 2006
    Location
    من ارض الكنانة
    Gender
    Female
    Religion
    Islam
    Posts
    27,759
    Threads
    1260
    Reputation
    246643
    Rep Power
    258
    Rep Ratio
    89
    Likes Ratio
    23

    The Medical student Review (OP)


    I don't know how many med students there are on board, or how useful this review will be for you.. I am a firm believer in slow and consistent than an intellectual enema two days before the exam where once released never again to be regained.. So what I plan to do here, is share some things I feel are important..

    In my pre-clinical years, I was president of the pharmacology club, and I enjoyed teaching it and exchanging ideas with others.. so if you have your own forte, quirks mnemonics that you'd like to share we can make a useful compendium..

    I am only going to focus here on pharmacology and diagnostic testings.. so every day I'll give you five of each..

    I am not going to start in any particular order but once in a section, I plan to complete it.. this will be just the high yield..


    __________________________________________________ ___________

    Oncology diagnostic testing
    _____________
    AFP (what disease is associated with it, and when do you answer this for a question?)
    AFP is associated with the development of
    1-hepatocellular ca.
    2-ovarian cancer
    3-non-seminomatous germ cell tumors
    Answer AFP when you see a patient with alcoholic cirrhosis or chronic hepatitis B or V. AFP together with radiological imaging is used to screen for hepatocellular ca

    ____________________________________________
    CEA
    1-A protein elevated in a variety of cancers including colorectal cancer
    2-serum CEA have a prognostic value in pts with newely diagnosed CRC. Those with higher levels have worse prognosis.
    3-CEA level to monitor in colon cancer in pts after a surgical resection, it determines the presence of persistent, recurrent or metastatic disease

    __________________________________________________ ___________
    Colposcopy
    Colposcopy is the direct visualization of the cervix, by use of a magnifying scope with a lamp
    2-The transition zone must be visualized to ensure an adequate colposcopy. The border between squamous and columnar epithelium
    3-a colposcopy is the answer for a pt with an abnromal pap
    Atypical squams can't exclude high grade lesion ASC-II
    low grade squamous intraepithelial lesions LSIL
    high grade intraepithelial lesions HSIL
    Atypical sqams of undetermined significance ASCUS if HPV DNA testing is positive
    __________________________________________________ ___________
    Estrogen and progesterone receptors
    Should be done on all pts with breast cancer, in order to determine who should receive hormone therapy
    2-Therapy with either tamoxifen or raloxifene should be added to any pt. with positive receptors. This is either for estrogen or progesterone positivity alone or in combination. The response to tamoxifen is better if both receptors are positive
    __________________________________________________ ____
    Mammogram
    screening should begin at age 40 and should be performed every 1~2 years, screening at age 50 should be yearly
    2-when mammogram shows abnormalities, a core biopsy including sentinel lymph node biopsy is the next best step. carcinomas of the breast are associated with clustered polymorphic microcalcifications.
    3-screening lowers mortality most after age 50 and the dec is greater than that of a colonscopy or a pap smear
    __________________________________________________ ________________

    Now pharm
    _____________

    will start with infectious disease because it is the longest chunk
    Acyclovir/valcyclovir/famiciclovir

    all the above are the correct answer for
    Herpes simplex including, genital, cutaneous, orolabial
    for Herpes Encephalitis (acyclovir) IV form only in a hospital setting
    Varicella zoster
    shingles: Herpes zoster or reactivation
    Bell's palsy

    the above meds work by inhibition of thymidine kinase
    -most common adverse effects are nephrotoxicity presumably from precipitation of the meds in the kidney tubule, sx of neurological toxicity in the kidney tubule, sx of neurological tox, such as confusion, tremors and hallucination occur rarely
    __________________________________________________ __
    Rifaximin
    used to treat travelers' diarrhea such as that from E.coli, it isn't used for invasive diarrhea. an associated fever and bloody diarrhea indicate and invasive pathogen, such as campylobacter. When fever and bloody diarrhea are described, ciprofloxacin is the best answer.

    rifa is a nonabsorbed version of the rifamycin antibiotic, it inhibits ribosomal RNA production of essential proteins
    There are no major side effects since it isn't absorbed from the GI tract. it doesn't cause C.Diff, and may in fact treat it.
    _________________________________________________
    Daptomycin and Linezolid

    they are both used for gram +ve organisms such as MRSA, streptococci and vanc resistant enterococci. Linezolid is the only oral antibiotic for MRSA. They can both be used for Vanc resistant organisms .

    Liezolid is an oxazolidinone and inhibits protein synthesis at the ribosome. Daptomycin is a cyclic lipopeptide and disrupts cell membrane they are both unique classes of drugs

    Linezolid commonly causes thrombocytopenia, and is a MAO inhibitor, avoid tyramine foods. Dapto caused CPK on liver functiion tests to be elevated!!
    __________________________________________________ _____________
    Tigecycline
    an extremely broad spectrum anti-biotic that covrers MRSA, staph aureus, and well as gram negative bacilli. Tigecycline is the answer for complicated hospital of ICU acquired infections, tigecycline alone is equivalent to vanc and aztreonam in combination, it is also active against resistant enterococci and PCN resistant penumococcus

    tigecycline is a glyclycyline antibiotic that binds to the ribosome and inhibits protein synthesis, it is unique that it covers staph, strep, gram negatives, anerobes and organisms resistant to vanc
    tigecycline is hepatotoxic , caused nausea and diarrhea
    __________________________________________________ ________

    Polymyxin B (PMB) and colistin
    useful for conjunctivitis, infections of the skin, and otitis externa, also correct for multi drug resistant gram negative bacilli, that cause ventilator associated pneumonia and sepsis from pseudomonas or acinetobacter.

    works by disrupting phospholipids in the cell wall membrane
    Polymyxin B (PMB) and colistin are very toxic to the kidney and nerves and is limited to topical applications of the skin and ear, they are also used for multi drug resistant gram negative bacilli when there are no other therapeutic options..

    __________________________________________________ ______
    That is it for today..
    The Medical student Review

    Text without context is pretext
    If your opponent is of choleric temperament, seek to irritate him 44845203 1 - The Medical student Review


  2. #21
    جوري's Avatar Full Member
    brightness_1
    Soldier Through It!
    star_rate star_rate star_rate star_rate star_rate star_rate star_rate star_rate star_rate star_rate star_rate
    Join Date
    Jul 2006
    Location
    من ارض الكنانة
    Gender
    Female
    Religion
    Islam
    Posts
    27,759
    Threads
    1260
    Rep Power
    258
    Rep Ratio
    89
    Likes Ratio
    23

    Re: The Medical student Review

    Report bad ads?

    Rheumatology pharm and on to tox.

    NSAIDS
    Naproxen
    Sulindac
    ibuprofen
    diclofenac
    etodolac
    indomethacin
    ketrolac
    piroxicam

    Cox-2 inhibitors:

    Rofecoxib
    celecoxib-- removed from market
    Valdecoxin (removed)

    NSAIDS and COX-1 inhibitors are indicated as analgesics.. also useful for the folowing

    Inflammatory syndromes , such gou, pseudogout, rheumatoid arthritis, and ankylosing spondylitis.
    cystic fibrosis
    fever
    still's disease
    Both NSAIDS and COX-2 inhibit prostaglandins

    NSAIDS cause peptic ulcer disease, and renal insufficiency, such as interstitial nehritis and nephrotic syndrome . Although COX-2 inhibitors have less effect on the gastic mucosa, they hae very severe cardiac toxicity. The COX-2 inhibitors rofecoxib, and valdecoxib were removed from the market because of excess cardiac deaths, only celecoxib remains on the market.

    ___________________________________________

    1 48 year old man comes to the ER with severe sudden pain in left knee after a beer binge
    over the weekend. on examination, he has a fever. The knee and toe are red and swollen. Joint aspiration shows 25,000 white cells that are predominantly neutrophils, and crystals are present. The crystals are needle shaped and negatively birefringent . creatinine is 2.4

    colchicine is the best initial therapy for acute attachs of gout, particularly when NSAIDS are contraindicated. In this case, a creatining elevation is contraindicated to NSAIDS. Colchicine is also used to treat familial mediterranean fever.

    Colchicine inhibits leukocyte mobility, decreasing the white cells ability to phagoctytose within the joint space and decrease lactic acid within the joint. This action reduces the deposition of the urate crystals that perpetuate the inflammatory response.

    the most common adverse effect of colchicine is diarrhea. in fact you should give colchicine to relieve pain until it produces diarrhea, rarely, colchicine may produce aplastic anemia.

    If there is no response to colchicine then the next therapy is intracrticular steroids.

    ______________________________________

    Allopurinol

    Allopurinol is a drug that lowers urate synthesis and ecreases the serum uric acid level.

    Allopurinol is a xanthine oxidase inhibitor. this reduces the uric acid level in both blood and urine.
    Answer allopurinol when the question describes a patient with recurrent gouty attacks, tophi, and uric acid stones and who has failed pobencid or sulfinpyrazone. The patient should be between attacks. Allopurinol has no benefit during acute attack of gout, because it is not anti-inflammtory.

    AAllopurinol is highly allergenic and can cause rash, eosiniphilia and interstitial nephritis.

    __________________________________

    Toxicology Pharm

    two drunk men come to the ER. one has visual disturbances, the other hyperemia of the retina. The other has developed kidney stones, he has envelope shaped crystals in his urine, both have a metabolic acdosis with an increased antion gap

    the first drunk with visual disturbance and an abnromal fundoscopic exam has methanol intoxication. The second patient who is drunk with oxalate crystals in the urine has ethylene glycol intoxication. oxalate crystals appear in the shape of an envelope. both cause metabolic acidosis.

    Fomepizole is the best initial therapy for ethylene glycol or mathanol poisoning. Fomepizole is considered superior to ethanol infusion, because it isn't intoxicating. Definitive therapy is with dialysis to remove the substances.

    Fomepizole inhibits alchol dehydrogenase. This prevents the production of the toxic metabolite and gives time for dialysis to be effective.

    __________________________________________

    32 year old woman comes to the ER dept. six hours after having ingested a bottle of 50 extra-strength (500 mg each)
    acetamiophen tablets

    best initial therapy for an overdose of acetominophen is acetyl-cysteine (NAC) and charcoal. Giving NAC is more important than getting a specific level of acetamiophen when the pt states she took a potentially harmful amount.

    NAC works by replacing the glutathione reductase that is depleted from metabolites of acetominophen. When glutathione reductase has been depelted, the liver cells start to necrose.

    charcoal can be used at the same time as the NAC. Charcoal doesn't bind enough of the NAC to result in a clinically significant impairement of its effect.

    __________________________________________


    Diagnostics still in heme

    Factor V leiden mutation

    factor V leiden predisposes to thrombosis by resistance to the antithrombotic effects of activated protein C. Protein C normally slows the clotting cascade by inhibiting factor V. the mutation allows factor V to ignore the natural anticogulant action of protein C. Factor V leiden i the most common cause of inherited thrombophilia.

    Answer factor V leiden mutation as the most accurate test in a young person with an unprovoked DVT or PE. Thrombotic events after plane flights should evoke and investigation for thrombophilia.

    the other tests of hypercogulable state are
    protein S
    Protein C when you see skin necrosis
    lupus anticoagulant when you see an elevated PTT or spontanous abortions in the case
    antithrombin III mutation, when you see resistance to heparin in the case.

    ________________________________

    philadelphi chromosome

    represents a genetic translocation between 9 and 22. it is associated with chronic mylogenous leukemia. It is also known as a BCR/Abl and can be detected by PCR.
    answer Philadelphia chromosome when presented with a case pf probable CML. the white cell count will be very high, mostly neutrophils, and the LAP score will be low. The Philadelphia chromosome also has prognostic value. If you give imitanib (geelvec) the philadelphia chromosome goes away the prognosis is good.

    __________________________________

    Haptoglobin levels

    used to determine hemolysis. It is a proten that binds to free hemoglobin. so when we ave hemolysis, RBCs will release free hemoglobin that will bind to haptoglobin. This will result in decreased haptoglobin levels. In hemolysis we also find elevated LDH, reticulocytes, and indirect bilirubin

    typical scenario is sudden anemia without Gi blees. the presence of Jaundice is also highly suggestive. Acute enamia minus the Gi bleeding equals hemolysis.

    ____________________________________________

    Hemoglobin electrophoresis,
    is the most sensitive test to diagnose hemoglobinopathis such as SCD or thalasemia. The most accurate way to diagnose the presence of the hetrozygou forms of these disease or the trait
    with respect to SCD, clinical presentations include ulcerations of the skin, of the legs, recurrent infections with pneumococcus or haemophilus, retionpathy, aseptic necrosis of the femoral head, osteomyelitis, growth retardation, and splenomegaly, Typically, the pt will be African American with a possible fam hx of the disease, Sickle cell trait will be in a pt who is asymptomatic with a family member with sickle cell disease or with unexplained hematuria.
    with respect to thalassemia, clinical rpesentations range from normal to severely symptomatic with growth failure, hepatomegaly, jaundice and bone deformities..

    ____________________________________

    Leukocyte alkaline phosphatase LAP score
    this is an enzyme in WBC. If the cells are elevated in number and the function os normal, the LAP score will go up in proportion to the elevated count.
    LAP is a test for CML, LAP white count is extremely hight and the differential shows mostly neutrophils. The case is likely to have a big spleen, giving upper quadrant pain and early satiety. LAP score should be LOW in CML, and are used to differentiate CML from leukemoid reaction..

    >>>>>>>>>>>>>>>>>>>>>>>>>
    The Medical student Review

    Text without context is pretext
    If your opponent is of choleric temperament, seek to irritate him 44845203 1 - The Medical student Review


  3. Report bad ads?
  4. #22
    جوري's Avatar Full Member
    brightness_1
    Soldier Through It!
    star_rate star_rate star_rate star_rate star_rate star_rate star_rate star_rate star_rate star_rate star_rate
    Join Date
    Jul 2006
    Location
    من ارض الكنانة
    Gender
    Female
    Religion
    Islam
    Posts
    27,759
    Threads
    1260
    Rep Power
    258
    Rep Ratio
    89
    Likes Ratio
    23

    Re: The Medical student Review

    ok today's 5/5..
    still on heme for diagnostics
    lymph node biopsy
    is used to detect infections such as TB, fungi and staphylococcus. Infections are suggested by nodes that are warm, tender and sometimes red.

    excisional lymph node biopsy is a single lymph node that is most accurate to diagnose a lymphoma. A needle biopsy is a wrong answer. you need to see architecture. In addition, the individual lymphocytes will appear normal on a needle biopsy. The diagnosis of lymphoma requires the visualization of the architecture of the entire node. Noses with lymphoma are nontender, not red, not warm as they usually are in an infection.
    _________________________________
    methylmalonic acid (MMA) builds up when vit B12 is deficient. MMA has greater sensitivity than Vit B12 levels. Homocystine is elevated in both B12 and folic acid deficiency.

    this is the test for a pt with macrocytic anemia and hypersegmented neutrophils but a normal B12 level.

    ______________________________________

    Mixing studies

    are done to distinguish between a clotting factor deficiency and an inhibitor of the clotting factor as the cause of an abnromal partial thromboplastin time (aPTT) you mix normal pooled plasma with the patient's plasma.
    this is the best test wen you have a pt with an abnromal APTT.
    if the test normalizes after mixingg, then the elevated aPTT is caused by a clotting factor deficiency.
    The next best test if the mixing normalizes is individual clotting factors assyas of the patient's plasma to determine which factor is deficient.
    if the test doesn't normalize it means that an inhibitor is present i.e factor VIII inhibitor of the lupus anticoagulant.

    __________________________________
    osmotic fragility test

    is the test for heriditary spherocytosis.
    it measures the amount of emoglobin released from red blood cells RBCs placed in hypotonic solution. RBC's with a reduced surface to volume ratio will lyse at concentrations that not affect normal cells. This is because they don't have enough cell membrane. They 'stretch' an then 'pop' when put into hypotonic solution.

    this is the test for a case of hemolytic anemia and splenomegaly with spherocytes on the peripheral smear. The other features that push the diagnosis and this test are:
    family history of recurrent episodes of anemia and bilirubin gallstones.

    ______________________________________
    Ristocetin cofactor assay.
    is a test for the function of von willebrand's VWF. Along with a VWF level. it is the most accurate test for Von willebrans's disease. Ristocetin acts and an artificial endothelial lining. if VWF is present and is functioning normally then platelets should adhere to Ristocetin.

    this is the test for a pt with platelet type bleeding and a normal platelet count and normal VWF level. Platelet type of bleeding is superficial. Examples are epistaxis, petechiae, purpura, and gum or gingival bleeding.

    _________________________________________

    in pharm, I will finish the last of tox and start cardio, like infectious disease cardio will be a large review..

    tox.

    70 year old woman with hx of osteoarthritis comes to the ER because of SOB, tinnitus, and decreased hearing. Her blood gas is initially alkalotic but changes to a ph of 7.28, wit a PCO2 of 23. Her Serum bicarbonate is low nd the anion gap is increased

    this is classic salicylate toxicity... pts presents with hyperventillation secondary to direct stimulation of the respiratory center in the brain. In addition, ASA tox can cause tinnitus, encephalopathy, and with severe toxicity, pulomary edema and yperthermia. In early tox there is respiratory alkalosis from direct stimulation of the brain, followed by metabolic acidosis, because ASA poisons the mitochondria and you lose te aerobic metabolism of oxidative phosphorylation. Lactic acidosis accumulates from anerobic glycolysis.

    bicarbonate is the best initial therapy for ASA poisoning.
    Bicarbonate will alkalinize the urine, resulting in increased excretion of ASAP at the kidney tubule
    _________________________________

    An alacoholic pt with seizure disorder is admitted with drowsiness, dysarthria, and difficulty walking. She does not know which meds she is on for her seizures. On. P.E there is ataxia ad vertical nystagmus. The head CT is normal

    phenytoin toxicity is the agent most likely to cause drowsiness, ataxia, and nysstagmus. The tye os nystagmus correlates with the severity of the phenytoin tox. Mild tox leads to horizontal nystagmus. in fact, horizontal nystagmus maybe present even at the upper end of normal therapeutic leve of phnytoin .. severe tox leads to vertical nystagmus.

    there is no specific antidote for severe tox.

    ________________________________

    a man with a HX of depression and an empty bottle of amitriptylene comes to the ER dept with obtundation, dry mouth, dizziness, flushing and dilated pupils.

    and EKG is the most urgent ste in the evaluation of tricyclic antidepressants overdorse. The EKG will show prolongation or widening of the QRA.
    the most common case of death from TCA are arrhythmias and seizures.

    TCAs inhibit the fast sodium channels in the His-Purkinje system as well the atrial and ventricular myocardium. this decreases conduction velocity, increases the duration of repolarization, and prolongs the absolute refractory periods. This effect is similar to that of quinidine.

    Bicarbonate should be given if the QRS prolongation is above 100millisecons.
    ________________________________________

    a guard is being evaluated after a nerve gas attack, he is found to be lacrimating, urinating and defecating, in addition he has respiratory distress and bradycardia


    Atropine is the best initial therapy for organophosphate poisoning. Atropine reverses the effect of the anti-cholinsterase inhibitor and the massive amount of acetylcholine that accumualtes. Organophosphates are absorbes through the skin. Further absorption is prevented for removing the patient's clothes and washing the patient.

    Pralidixime is the definitive treatment for reversing organphosphate toxicity. pralidoxime reactivates acetylcholinesterase.
    ______________________________________________

    Angiotensin-Converting Enzyme (ACE) inhibitors:

    Ramipril, Lisinopril, Fosinopril, Enalapril, Perindopril, Captopril, Trandolopril
    ACE inhibitors are indicated for the following conditions:

    Congestive heart failure with low ejection fraction (prevents left ventricular (LV) remodeling
    hypertension (particularly in diabetics)
    proteinurea, including microalbuminurea
    Acute myocardial infarction prevents worsening LV function
    hypertensive crisis

    Ace inhibitors block the production of angiotensin II (AT II) in the lung. At II is a potent vasconstrictor. AT II also stimulates the release of aldosterone from the zona glomerulosa of the adrenal gland.

    The most common adverse effect of ACE inhibitors is a dry cough. Angioedema and hyperkalemia may also occur. Ace inhibtors transiently decrease the gloerular filtration rate (GFR) but are renal protective in long term.

    >>>>>>>>>>>>
    The Medical student Review

    Text without context is pretext
    If your opponent is of choleric temperament, seek to irritate him 44845203 1 - The Medical student Review


  5. #23
    جوري's Avatar Full Member
    brightness_1
    Soldier Through It!
    star_rate star_rate star_rate star_rate star_rate star_rate star_rate star_rate star_rate star_rate star_rate
    Join Date
    Jul 2006
    Location
    من ارض الكنانة
    Gender
    Female
    Religion
    Islam
    Posts
    27,759
    Threads
    1260
    Rep Power
    258
    Rep Ratio
    89
    Likes Ratio
    23

    Re: The Medical student Review

    Still on heme in diagnostics

    Russel's Viper Venom Clotting Time (RVVT)

    RVVT is a phospholipid-dependent coagulation test.
    it is used in detection of antiphospholipid antibodies or lupus anticoagulant. Think of this in a patient who has a prolonged PTT that doesn't correct with a mixing study. The dilute Russell viper Venom test maybe indicated to confirm that the inhibitor is a lupus anticoagulant.

    Clinical scenario may be of a woman with or without features of SLE, with repeated abortions or giving birth to an infant with heart block or presenting with venous or arterial thrombosis.

    _________________________________________

    Shilling's test

    is ded to find the etiology of a B12 deficiency. The most common cause of B12 deficiency is pernicious anemia, where there is decreased intrinsic factor due to antibodies against IF

    _________________________________

    in the first stage, radiolabeled cyanocobalamine is given orally, followed by an intramuscular injection of cyancobalamine 1 hour later, urine is collected for determination of the percent excretion of the oral dose. In pernicious anemia, or malabsorption, excretion in the urine is low. The test is repeate with added oral intrinsic factor, Adding IF should normalize cyanocobalamine absorption and urinary excretion in pts with pernicious anemia, but not in those with intenstinal malabsorption.
    schillings's test is hardly ever used, except in cases with dx of B12 deficiency where the diagnosis isn't pernicious anemia..

    _________________________________________________

    Serum protein electrophoresis (SPEP)

    the SPEP is the separation and fractionation of the pt serum proteins in order to assess the individual component, i.e how much os the protei is albumin how much is mmunoglobulin

    SPEP is used to evaluate a high total serum protein. SPEP is the best initial test to diagnose myeloma, particularly if an x-ray of the bone looking for lytic lesions has already been done, or is x-ray is not one of the answer choices.
    the most common reason to have an abnormality on the SPEP is a monocolonal gammopathy of unknown significance.
    the most accurate test for an IgG abnromality detected as an SPEP spike is a bone marrow. A total o >10% plasma cells is indicative of myeloma. Waldenstrom's macroglobulinemia will also give an abnromal SPEP with an IgM spike.

    __________________________________________________ __

    Sucrose lysis test
    is a screening test for paroxysmal nocturnal hemoglobinuria (PNH) a sample of the patient's blood is tested with sucrose and observed for evidence of hemolysis from complement activation. The hemolysis of PNH is caused by increased sensitivity of the patient's red blood cells to lysis by complement and evoked hemolysis.
    sucrose lysis is key when the t presents with dark urine in the morning, with or without evidence of venous thrombosis and pancytopenia. The coomb's test will be negative,

    the most accurate test for PNH is flow cytometry for the presence of CD55 and CD59, also known as decay accelerating factor. (DAF)

    _________________________________________

    last is on opthalmology

    edema - The Medical student Review

    this is a photograph of proliferative diabetic retinopathy (PDR). There is neovascularization over the disc, covering much of the surface area of the retina. Neovascularization differentuates PDR from non PDR. the aberrant blood vessles are fragile and leaky causing recureent retinal and vitreous hemorrhages. Dot blot hemorrhages, micoaneurysms, hard exudates, cotton wool spots, and intraretinal microvascular abnromalities (IRMA) are present in both PDR and none PDR

    PDR can lead to retinal detachment and neovascular glaucoma
    the treatment is with pan retinal photocoagulation.
    ___________________________________________

    Now Pharm cardiac
    ___________

    64 year old woman with a HX of HTN comes to the ER dept with palpitations, she is found to have atrial fibrillation and pulse rate of 125/mins

    atrial fibrillation and atrial flutter with a rapid ventricular rate are best treated with either a calcium channel blocker, such as verapmil or diltiazem, a beta blocker such as metoprolol or digoxin. The target rate is less than 100 BPM

    calcium channel blockers cause hypotension, constipation, peripheral edema and heart block.
    beta blockers, cause hypotension, bronchospasm, depression, erectile dysfunction and worsening dyslipidemia
    dignoxin causes dysrhytmia, hyerkalemia, confusion, diarrhea and visual changes.
    chronic atrial arrhythmias should be treated with coumadin with an international normalized ratio (INR) of 2-3 to avoid an embolic stroke.

    Electrical cardioversion is indicated when the patient is hemodynamically unstable, exhibiting such symtoms as hypotension, hemodynamically related confusion, shortness of breath or chest pain.

    _______________________________________

    calcium channel Blockers
    Diltiazem, Verapamil, Nifedipine, Felodipine, Nicardipine, Amlodipine, Nitrendiine, Nislodipine, Isradipine
    Calcium channel blockers are correct for the following conditions
    HTN (in pts with diabetes or high risk coronary disease)
    Atrial arrhythmias
    ulmonary hypertension
    Hypertrophic cardiomyopathy
    Raynaud's phenomena
    Subarrachnoid hemorrhage

    CCB's work by causing vasodilation by relaxing smooth muscle in the vascular lining. CCB's also inhibit conduction in the AV node of the heart
    All CCB's cause postural hypotension, flushing, constipation, and edema, diltiazem and verapamil can cause AV block. The others particularly nifedipine can cause tachycardia.
    ___________________________________________

    Spironolcatone/Eplerenone
    Maybe appropriate treatment for the following conditions
    congestive heart failure (CHF): as one of the medication that will lower mortality. Eplerenone is an alternative for CHF only
    Ascites: as the best initial diuretic therapy
    Acne, especially for women because of its anti-androgenic qualities
    HTN-rarely
    Amenorrhea
    Adrenal hyperplasia or aldosterone producing adenomas
    this is best for CHF however along with ACE inhibitors and beta blockers.

    Spirinolactone is an aldosterone antagonist, it has antiandrogenic effects, which is why it helps with hirsutism, acne and amenorrhea. but it can also cause gynecomastia and hyperkalemia.
    Eplerenone is used for CHF and HTN as well but does not inhibit testosterone receptors, so it can cause hyperkalemia but not gynecomastia

    ________________________________________

    Propanolol

    is indicated for the following conditions
    Cluster migraine headache prophylaxis: Must be taken for several weeks to prevent headaches
    Portal HTN, decreases the frequency of bleeding from esophageal varices.
    Thyroid storm, propanolol decreases symptoms acutely
    Essential tremor
    pheochromocytoma

    propanolol is a nonspecific blocker of beta -1 and beta -2 receptors
    associated with worst side effect profile than beta one specific, such as atenelol, metoprolol. propanolol can cause bronchospasm and depression, bradycardia and hypotension and erectile dysfunction. it can also cause hyperkalemia by inhibiting sodium/potassium Atpase. Propanolol can have adverse effects on glucose and peripheral arterial disease which rarely occurs with selective blockers.

    ________________________________________

    34 year old woman comes to the ER with palpitation. An EKG shows supraventricular tachycardia at a rate of 160 min, there is no response to vagal maneuvers, such as carotid sinus massage.

    adenosine is the drug of choice for SVT that is not responsive to vagal tone carotid sinus massage.
    Adensine reduced calcium currents and is antiarrhythmic by increasing AV nodal refractoriness. It transiently slows the sinus rate and the AV nodal conduction velocity. It is thought to open potassium channels, hyperpolarizing nodal tissue and making it less likely to fire.
    Adenosine causes transient asystole, but this usually lasts less than 5 seconds, a bolus can precipitate a bronchospasm.

    >>>>>>>>>>>>>>>>>>>>>>>>>>>>
    The Medical student Review

    Text without context is pretext
    If your opponent is of choleric temperament, seek to irritate him 44845203 1 - The Medical student Review


  6. #24
    جوري's Avatar Full Member
    brightness_1
    Soldier Through It!
    star_rate star_rate star_rate star_rate star_rate star_rate star_rate star_rate star_rate star_rate star_rate
    Join Date
    Jul 2006
    Location
    من ارض الكنانة
    Gender
    Female
    Religion
    Islam
    Posts
    27,759
    Threads
    1260
    Rep Power
    258
    Rep Ratio
    89
    Likes Ratio
    23

    Re: The Medical student Review

    I am going through a rough time folks wal7mdllah, so If you are reading this, pls remember me in your du3a

    ok.. on with opthalmology

    photo336 - The Medical student Review

    this isn't a clip from the incredible hulk, it is fluorescein stain of cornea

    after manul staining with fluorescein, the eye is exposed to blue light tht detects foreign bodies or damage to the cornea. fluorescein staining i used in the evaluation of the following conditions
    Corneal Abrasion
    infection
    injury or trauma
    foreign bodies
    abnormal tear production
    kertoconjunctivitis sicca
    abnromalities in the surface of the cornea will be stained and appear green
    normally the dye stays in the tear film (water, mucus, oil) and doesn't adhere to the cornea
    the most likely question in which fluorescein stain is the answer, is whe you are shown a person who had an ocular truama and there is severe pain, and you need to exclude corneal abrasion .
    _______________________________________________

    retinal20detachmentJPG 1 - The Medical student Review
    pts with retinal detachment often describe an ascending/descending vein in front of one eye with flashes, floaters. The unilateral loss of vision is sudden and painless.
    retinal detachment is usually caused by a retinal tear
    retinal detachment is a medical emergency. Laser surgery or cryoplexy is always used to help reattach the retina. A scleral buckle may be placed like a belt around the outside of the eye to push the wall against the detached retina, or a vitrectomy may be done where the vitreous is removed and replaced with a gas that pushes the retina against the wall of the eye.

    ______________________________________

    rothspotwhitecenteredhemorrhageendocardi 1 - The Medical student Review

    Roth spot is actually a cotton wool spot (infarct) with surrpunding hemorrhage.
    Subacute bacterial endocarditis is commonly associated; however, roth sports are not specific for subacute bacterial endocarditis, and maybe present in other disorders, such as diabetes, Leukemia, anemia, hypertension and HIV

    Roth spots in subacute bacterial endocarditis are an immunological phenomenon.

    _______________________________________

    artmn442145reimfig5 1 - The Medical student Review

    slit lamp exam is a microscopic examination of the anterior half of the eye by projecting a thin beam of light through a slit. The anterior half includes lids, lashes, adnexa, conjunctiva, sclera, cornea, iris, pupil, and lens. When combines with special lenses, the slit lamp may also be used to examine the posterior eye (cup/disc, macula, vessles, periphery)

    in this photo
    kf2 1 - The Medical student Review

    you see kayser fleischer ring in Descemet's membrane on slit lamp, which are basically copper deposits. This finding is diagnostic of Wilson's disease, a disorder of copper excretion.

    Slit lamp is the test for a pt. with red painful eye with pain worsened by shinning a light in the eye or photophobia. When severe photophobia isn't associated with meningitis, it can be from iritis or uveitis, which is an inflammation of the iris. Slit lamp is also a good way to detect cataracts or corneal injury.

    ___________________________________________

    CZM June A03 Fig05 - The Medical student Review

    Gonioscopy is visualization of the angel between the cornea and the iris. This is performed to assess for narrow angle or angle closure in glaucoma

    in Gonioscopy, light from the slit lamp is deflected obliquely through the cornea using a prism or mirror in order to get a view between the cornea and iris. The trabecular mshwork and schlemm's canal, the drainage system of the eye, lie in thi angle.

    Answer Gonioscopy when you see a case of glucoma with a red, painful eye with fixed, mid dilated pupil and tonometry has already confirmed glaucoma. Gonioscopy is used to determine the types of glaucoma after the diagnosis has been made.

    ________________________________________

    now cardio in pharm

    ________________

    Aspirin
    used for the following

    Acute coronary syndromes (myocardial infarction \[MI} and unstable angina as well as post stent and post surgical bypass pts.
    Stroke and transient ishchemic attack (TIA)
    Peripheral arterial disease
    fever reudction (antipyretic)
    rheumatoid arthritis
    essential thrombocytopenia
    kawasaki's disease
    Arthrtis, gout and in general as an analgesic

    Asa irreversibly inhibits platelets b inhibiting cycloxygenase
    the most common adverse effect is bleeding. Aspirin also causes peptic ulcers, asthma, renal insufficiency and rash. In toxic amouns, aspirin leads to metabolic acidosis, tinnitus, encephalopathy, renal insufficiency, and increased anion gap.

    ________________________________________________

    Dipyridamole

    is never used as first line therapy for anything.
    but can be used for the following conditions
    Stroke in combination with Aspirin
    preventing heart embolic complications
    peripheral arterial disease with asa
    as diagnostic testing in myocardial perfusion stuies with thallium

    dipyridamole inhibits adenosine deaminase and phosphodiesterase, which increase levels of cyclic adnosine monophosphate (cAMP) cyclic AMP inhibits platelets
    adverse effects can include the following
    dizziness and headache
    gastrointenstinal bleed
    ____________________________________________

    Clopidogrel and ticlopidine
    are not the best initial therapy for anythng, clopidogrel is the right choice after ASA can't be tolerated or has faled. clopidogrel is used for acute coronary syndromes, stroke, peripheral vascular disease and post stent placement in combination with ASA

    clopidogrel and ticlopidine inhibit platelet aggregation, they prevent adenosine diphosphate (ADP) from stimulating platelet and fibrinogen binding

    the most common adverse effect for both is bleeding, Ticlopidine is always the wrong answer therapeutically. Ticlopidine is most often associated with neutropenia and thrombotic thrombocytopenic purpura.

    ++++++++++++++++++++++++++++++++++++++++++++++++++ +++

    Cliostazol
    is a phosphodiesterase inhibitor that is used or peripheral arterial disease
    cilostazole increases levels of cyclic adenosine monophosphate (cAMP) it irreversibly inhibits platelet aggregation by inhibiting thrombin, adenosine diphosphate (ADP), collagen and epinephrine. It is also a vasodilator that is greatest in the femola bed and less in the vertebral carotid and mesenteric arteries.

    a patient with intermittent claudicaton from vascular disease should be treated with cilostazole. It is the single best therapy for PAD. It is used in addition to ASA, dipyridamle and exercise.
    adverse
    it canses edema, dizziness, and vertigo. The most serious effect is Afib, ventricular tachycardia and CHF.

    >>>>>>>>>>>>>>>>>>>>>>>>>>>>>>>>>>>>>>>>>

    73 year old man with CHF recently started onramipril, metoprolol and spirinolactone, and furosamide. He has developed chronic dry cough that makes it difficult for him to sleep..

    ACE are the most likely medication to be causing chronic cough, the sx is secondary to their effect on bradykinin levels

    Angiotensin receptor blockers (ARBs) should be started in pts who are intolerant of ACE inhibitors. ARB's have the same indications as ASC, such as CHF, HTN, and acute mycordial infarctions. They seem to have an equivalent mortality benefit as well. ARBs include losartan, olmessartan, valsartan, irbesartan, candesartan, telmisartan and eprosartan.
    The most common side effect o ARBs are hypotension and hyperkalemia.
    _______________________________
    Last edited by جوري; 09-26-2008 at 02:59 AM.
    The Medical student Review

    Text without context is pretext
    If your opponent is of choleric temperament, seek to irritate him 44845203 1 - The Medical student Review


  7. Report bad ads?
  8. #25
    جوري's Avatar Full Member
    brightness_1
    Soldier Through It!
    star_rate star_rate star_rate star_rate star_rate star_rate star_rate star_rate star_rate star_rate star_rate
    Join Date
    Jul 2006
    Location
    من ارض الكنانة
    Gender
    Female
    Religion
    Islam
    Posts
    27,759
    Threads
    1260
    Rep Power
    258
    Rep Ratio
    89
    Likes Ratio
    23

    Re: The Medical student Review

    ok.. still on cardio today and almost done with opthalmology we'll be moving on to other endeavors

    Pharm cardio
    _____________
    Glycoprotein IIb/IIIa
    inhibitors (GPIIb/IIIa)

    abciximab
    tirofiban
    eptifibitide
    lamifiban
    Orbofiban

    Glycoprotein IIb/IIIa inhibitors (GPIIb/IIIa) are platelet aggregation inhibitors that are useful in keeping the coronary artery open in acute setting

    Glycoprotein IIb/IIIa work by reversibly antagonizing the IIb/IIIa receptor on the platelet. This prevents fibringoen and Von Willebrand's factor from binding to the receptor on the platelet and thus, prevents the platelet from binding to the endothelial lining of each other.

    Glycoprotein IIb/IIIa inhibitors are an adjunctive therapy for angioplasty and other percutaenous coronary interventions. They are also useful for non-ST segment elevation myocardial infarctions when thrombolytics are not indicated.

    Glycoprotein IIb/IIIa inhibitors (GPIIb/IIIa) can cause bleeding, thrombocytopenia, and coronary artery dissection.

    _____________________________________________

    Thrombolytics
    tPA
    Anistreplase
    streptokinase
    Alteplase
    Tenecteplase

    Thrombolytics are recombinant versions of tissue plasminogen activator tPA that are used to reopen acutely thrombosed coronary arteries.
    tPA cleaves plasminogen to plasmin. plasmin will dissolve fibrin that has been freshly deposites. After several hour, fibrin is cross linked by factory XIII or 'clot stablizing factor' this makes fibrin refractory to dissolution by plasmin.

    Thrombolytics are contraindicated when there is a major bleed occuring. Such as melena or intracranial bleeding. It is also contraindicated with aortic dissection, head trauma or BP > 180/110

    tPA is the answer when the question describes chest pain within 12 hrs and a 1mm St elevation of a new left bundle branch block. tPA has less efficacy than primary angioplast for an acute infarction. tPA is also indicated for ischemic heart disease within 3 hours as well as pulomnary emboli with hemodynamic instability, such as hypotension.
    ___________________________________________

    Digoxin

    dig has two main indications. rate control of Afib and symptomatic control of CHF
    Dig DOESN'T LOWER MORTALITY, in CHF but it does decrease the severity of sx and frequency of hospitalization. dig will not convert afib to sinus rhythm, but it will slow the rate.

    dig inhibits the Na/K Atpase. this increases cystolic calcium levels and increases the force of contraction of cardiac muscle..

    dig at toxic levels can cause nausea, vomiting, arrhythmias, confuscion, hyperkalemia and visual disturbances.
    ________________________________________

    a pt with CHF is maintained only on a diuretic and digoxin is admitted because of confusion, hypotension, nausea, bradycardia and visual disturbances. His dix levels is markedly elevated


    Severe dg toxicity should be treated with dig binding antibodies (digibind) the indications for dig specific fab fragments (d-fab) are hyperkalemia, arrhythmias, encephalopathy, or hypotension.

    The most common electroradiographic abnromality is ectopy, hwever the most common serious rhythm disturbance is atrial tachycardia with variable block. Sinus bradycardia may occur.

    dig toxicity may lead to hyperhalemia because dig inhibits the Na+/K Atpase.

    ____________________________________

    Amiodrone

    is a potent antiarrhythmic medication with excellent efficacy but multiple side effects from long term use. It is the drug of choice for venticular fibrillation in an acute resuscitation.
    Amiodarone is structurally similar to thyroid hormone. It blocks inactivated Na+ and ca 2+ channels and has a beta blocking effect. It potently inhibits abnromal automaticity.

    Adeverse effecs are
    Hyperthyroidism, hypothyroidism, pulmonary fibrosis and non sight threatening corneal deposits.

    this is the DOC for ventricular fib and ventricular tachy
    treating a fib with systolic dysfunction


    ____________________________________

    Diagnostics

    Tonometry

    this is the test to measure intraocular pressure. it tests for glaucoma

    there are several ways of performing tonometry: pneumotonometry is used as a screening tool. It measures IOP with a puff of air to flatten the cornea. Golmann tonometry is the more accurate way to measure IOP. It combines a slit lamp with a special probe that flattens the cornea and a weight that determines the amount of pressure

    the most accurate test is electronic indentation tonometry where a small pinlike instrument is laces directly on the cornea and the pressure is read back on a monitor

    it is the best initial and most accurate test for glaucoma .

    ______________________________

    all right we are now up to infectious disease in diagnostics
    trichomonasVaginalis - The Medical student Review


    trichvag dk1 - The Medical student Review

    Trichomonas vaginitis


    this is treated with oral metronidazole with concurrent treatment of sexual partner. Single dose therapy is perferred because of increased pt adherence

    The pelvic exam will show a strawberry cervix which is cause by multiple punctate hemorrhages visible on cervix

    trichomonas is the only organism known to be mobile on wet mount, as the flagealla has it swim across the slide.

    _________________________________________

    alright the other three after I come back from a break
    ___________________
    aight back from break

    035 01 - The Medical student Review

    this is s india ink of the cereberal spinal fluid showing cryptococcus neoforman. The organism has a mucopolysaccharide capsule that provides a 'halo' around it that pushes away the india ink.
    this is the best initial test for cryptococcus neoforman, when you see a pt with AIDS with <100 CD4 cells who presents with fever, headache, and possibly a stidd neck. CSF will show a mild elevation of lymphocyts. The presentation is a 'subacute' which means it is slower in onset and milder than bacterial meningitis. Caution: Normal CSF protein and white cell counts do not exclude cryptococcal meningitis.

    The most accurate test for cryptococcal meningitis is either a cryptococcal antigen or fungal culture of the CSF.

    __________________________________________________ _______________

    artiiu431453fig1 1 - The Medical student Review

    the smear is a wet mount and shows clue cells. they are abnormal epithelial cells with ground glass appearance and a hazy border. The tiny dark dots are bacteria stick to the surface of the epithelial cells.

    there will be a thin discharge and itching, also dyspareunua. fishy odor positive for the whiff test adding KOH to slide. The PH is usually > 4.5
    pregnant women with BV are at an increased risk for miscarriage, early preterm delivery and infection after the pregnancy.

    ++++++++++++++++++++++++++++++++++++++++++++
    da 035470 - The Medical student Review
    ah the kit for a bone biopsy
    look for a pt with peripheral vascular disease, with leg pain, warmth and an ulcer, wo has an abnromality on x-ray or MRI consistent with osteomyelitis.

    needle aspirate is gold standard
    for
    osteomyelitis
    bone tumors
    metastatic bone disease

    radiological evidence of osteomyelitis and positive blood cultures can replace the bone biopsy if osteomyelitis is suspected.

    >>>>>>>>>>>>>>>>>>>>>>>>>>>>>>>
    Last edited by جوري; 10-12-2008 at 04:37 PM.
    The Medical student Review

    Text without context is pretext
    If your opponent is of choleric temperament, seek to irritate him 44845203 1 - The Medical student Review


  9. #26
    جوري's Avatar Full Member
    brightness_1
    Soldier Through It!
    star_rate star_rate star_rate star_rate star_rate star_rate star_rate star_rate star_rate star_rate star_rate
    Join Date
    Jul 2006
    Location
    من ارض الكنانة
    Gender
    Female
    Religion
    Islam
    Posts
    27,759
    Threads
    1260
    Rep Power
    258
    Rep Ratio
    89
    Likes Ratio
    23

    Re: The Medical student Review

    on with pharm
    _________________

    68 year old woman is being evaluated in your office for worsening sob with minimal exertion. her echo shows ejection fraction of 32%

    The medications that have a clear mortality benefit in CHF are as follows

    beta blockers, metoprolol, and carvedilol
    ACE
    ARBs candasartan and valsartan
    spironolactone

    An implantable cardioverter defeibrillator has benefit for reducing mortality with ishcemic cardiomyopathy and an ejection fraction <35%

    ____________________________

    27 year old woman from ecuador is being evaluated because of increasing shortness of breath. she shows rales on exam. she has an early diastolic decrescendo murmur. The symptoms have become worst because of pregnancy

    the patient has mitral stenosis
    mitral stenosis is most often from rheumatic fever in the past.

    best initial therapy for mitral stenosis is pre-load reduction with sodium restriction and a diuretic. This is problematic however, because of her pregnancy. Diuresis can potentially cause IUGR.
    The most effective therapy for mitral stenosis is baloon valvuloplasty. this procedure is ideal for a pregnant woman. pregnancy increases plasma volume by 50% and worsens sx. baloon valvuloplasty is safe in pregnancy and is by far preferable to open heart surgery.

    ____________________________________________

    Your pt has recently been diagnosed with coronary artery disease by stress test. He is a nonsmoker, he doesnt have diabetes or htn, his LDL is 145 after three months of life style modifications. including diet, exercise and attempts at weight loss

    the best initial therapy for high levels of LDL despite life style modification is a statn. LDL above 130 needs drug therapy.

    adverse effects: hepatotxicity, LFT's should be checked for thabdomyolysis or myositis though neither is as common as liver tox.
    The goal therapy for CAD is an LDL <100.

    ________________________________________________

    a diabetic hypertensive obese smoker is found on angiography to have CAD, her LDL is 110

    a diabetic is found to have an LDL of 122

    a man with CAD has an LDL of 170 despite rx with a statin and life style modifications for the last 6 months. his triglcerides are elevated and his HDL is low

    the best therapy initially is always a statin

    the goal therapy for a person with CA and diabetes or CAD and multiple risk factors is an LDL <70. established risks are diabetes, smokin, htn, and an age >45 in men and >55 in women

    case one statin
    2 the goal in a diabetic is an LDL <100

    when the goal can't be met with a statin, seconf medication should be added, cholestyramine bind cholesterol in the bowel but leads to bloating, abdominal pain and flatus. The best therapy specifically for triglycerides is a fibric acid derviative such as gemfibrozol.

    Niacin is the best med to raise HDL. Niacin causes flushing and elevations of glucose and uric acid. Ezeetamibe lowers levels but has no proven mortality benefit.

    the goal is an LDL < 100. the for for triglycerides is <150

    _________________________________________

    53 year old experiences syncope while at the opera. she loses her pulse, she is found to have ventricular tachycardia. Electrical cardioverson restores her to sinus rhythm

    an implantable cardioverter/defib is the best initial therapy to prevent sudden death from either ventricular tachycardia or ventricular fibrillation. Electrophysiologic studies aren't necessary. she has already sustained v tach with loss of pulse. inducing an arrhythmia is not necessary. Beta blockers may be helpful to prevent the arrhythmia from developing but the life threatening nature of this rhythm disturbance makes having a defirillator essential.

    _______________________________________________


    infectious dz in diagnostics

    Bone scan or technetium pyrophosphate nuclear bone scan

    Bone scan is a method of detecting occult disease that has deposited in bone

    technetium is picked up by osteoblasts and deposited in the bone as they lay down new matrix. You must have osteoblastic activity in order for it to light up

    bone scans can be abnromal from both infections and malignancies and cancer are detected by bone scans. Bone scans lack specificity and often can't distinguish soft tissue infections from nearby involvement of the bone.

    The rpecise etiology of the abnormality on a bone scan often requires a bone biopsy to confirm dianosis

    ____________________________________

    CD4 (t-Cell) counts

    CD4 count is used to monitor HIV positive pts. serum measurement of CD4 (t-Cell) markers is representative of the number of helper T lymphocytes in the blood count. CD4 is used to monitor the response to reatement. start antiretroviral therapy: CD4 <350 or viral load >55,000.
    answer CD4 count as the best test to determine the severity of immunosuppression in HIV/AIDS

    prophylaxis should be initiated in the following scenarios

    CD4 count less than 200 cell/ul: TMP/SMX for pneumocystis jirovrci
    CD4 count less than 100 cells/ul: TMP/SMX for toxoplasma
    CD4 counts less than 100 cells/ul: itraconazole for histoplasmosis in endemic areas.

    CD4 counts less than 50 cells/ul: azithromycin for mycobacterium avium complex (MAC)

    ___________________________

    will finish the last three after a break insha'Allah
    _________________________

    Darkfield microscopy

    the most accurate test for primary syphilis
    a scraping of swab from the syphilitic cancre is rubbed on a slide

    answer darkfield microscopy for primary syphilis when the questions asks for most accurate test. primary syphilis there is a false negative rate of about 25% for the RPR or VDRL. The darkfield is more sensitive than an RPR
    ___________________________________
    Gallium scanning

    is a nuclear scan that detects infection and some cancers. Gallium builds up in areas where white blood cells are present and there is increased iron metabolism. Gallium is transported on transferrin. Gallium is a nonspecific test that can help localize the site for subsequent CT or MRI scanning

    Gallium is considered on pts with persistent fever without localizing symptoms and the initial blood cultures, chest x-ray and urinalysis are negative. Gallium detects the source of fever os unknown origin, lymphoma, and abscesses.

    Ultimately, a biopsy is the most accurate way to determine the etiology of an abnromal gallium scan. Cancers require histology and infections require culture to be confirmed.
    __________________________________________

    herpes simplex PCR
    is done on CSF and is the single most accurate test for herpes encepalitis.

    look for a pt with fever, headache, and confusion of less than 2 weeks duration. The PCR of CSF is more accurate than an EEG, head CT, MRi or even brain biopsy.

    >>>>>>>>>>>>>>>>>
    Last edited by جوري; 09-28-2008 at 03:57 AM.
    The Medical student Review

    Text without context is pretext
    If your opponent is of choleric temperament, seek to irritate him 44845203 1 - The Medical student Review


  10. #27
    جوري's Avatar Full Member
    brightness_1
    Soldier Through It!
    star_rate star_rate star_rate star_rate star_rate star_rate star_rate star_rate star_rate star_rate star_rate
    Join Date
    Jul 2006
    Location
    من ارض الكنانة
    Gender
    Female
    Religion
    Islam
    Posts
    27,759
    Threads
    1260
    Rep Power
    258
    Rep Ratio
    89
    Likes Ratio
    23

    Re: The Medical student Review

    today's 5/5

    7904r fig2 - The Medical student Review

    indium labeled leukocyte scan
    is a nuclear medicine test of occult infection or FUO. A sample of the pt's blood is drawn and incubated with indium. The indium tags the white blood cells then localize to the site of infection.

    look for a case of occult infection not found on routine testing such as blood cultures, chest x-ray, or urinalysis. Indium is useful when gallium is inaccurate such as with intrabdominal infections.

    a biopsy is more accurate than an indium scan.

    _______________________________________

    9412147CandidaalbicansinKOHprep - The Medical student Review

    this is a KOH prep of candida
    is is the best initial test to diagnose superficial fungus. KOH is applied to a wet mount samle of vaginal discharge/secretions or to a skin scraping to identify the fungus. This is seen under the microscope. Epithelial cells dissolve an the fungal structures remain behind and are visible

    KOH is used for the following

    Tinea (pedis, manus, corporis, cruris, capitis)
    onchomycosis
    tinea versicolor
    candidiaisis

    applying KOH to a wet mount and smelling/whiffing immediately after yeilds fishy odor (amine) is bacterial vaginosis.

    _______________________________________________

    006 - The Medical student Review

    Monospot test is the initial test used to diagnose infectious mono .. it detects hetrophile antibodies that are charcteristic of Epstein barr virus

    monospot is the best initial test for a pt with a sore throat, fever, lymphadenopathy, and malaise, a total of 50% of pts will have splenomegaly, rash is present in 15% unless they have been given ampicilin. When it is present closer to 90%. Exudative pharyngitis may occur

    the most accurate test acutel is the IgM to EBV viral capsid antigen (VCA). antibodies to EBV nuclear antigen (EBNA) rise in 3-4 weeks
    _______________________________________

    1342 binax now legionella kit - The Medical student Review

    legionella urine antigen is found in legionella pneumonia. it is the best initial test
    legionella urine antigen has a 100% specificity with L. pneumophila type i. Only 70-80% of disease is caused by L- pneumoophila type 1. sputum culture or a culture of the tracheal aspirate on specialized charcoal/ yeast extract remains definitive.

    suspect legionella antigen when seeing a nursing home resident admitted for pneumonia that presents with fever, confusion, diarhhea and hyponatremia. CPK, liver function test and creatinine can also be elavated

    _________________________________________

    pcr1 - The Medical student Review.

    polymerase chain reaction HIV RNA viral load

    the PCR HIV viral load is a quantitative measurement of the amount of virus circulating in the pt's blood

    this test is the first thing to change in response to treatment. PCR HIV viral load is also the first thing to become abnromal is the patient stops taking medications or the treatment fails. The viral load test tells how fast the CD-4 T-elper cells will drop. The higher the viral load the faster the disease process.

    measurements should be made at the time of the HIV diagnosis and every 3-4 months thereafter. The PCR viral load is also the basis of HIV sensitivity testing in order to determine the presence of resistance in failing regimen.

    ________________________________________

    Now Pharm, finish cardio and move on to endocrine

    _______________________

    A man with metastatic cancer develops DVT/ woman with metallic heart valve presents for routine care.

    those with deep venous thrombosis (DVT) can receive either low molecular weght heparin or IV-unfractionated heparin, their efficacy is identical. The measurement is followed by warfarin (coumadin) to international normalized ratio (INR) 2-3

    heparin potentiates the effect of antithrombin on the clotting cascade

    those with a first DVT should be maintained on warfarin for at least 6 months

    most common adverse effect of both therapies is bleeding. Haparin can result in thrombocytopenia

    metal heart valves maintain lifelong coagulation with warfarin. pts with metal heart valves are the only pts n which you routinely maintain target INR above 2-3. the target INR is 2.5-3.5
    warfarin inhibits vitamin K dependent clotting factors (II,III,VII,IX and X)

    __________________________________

    a 32 year old woman in her third trimester with HTN, edema and proteinurea, se is being prepared for dleivery.

    67 year old man develops torsades de pointes post infarction, he is hemodynamically stable.

    Magnesium sulfate (Mg2+) is the best medical therapy for pre-eclampsia and eclampsia as well torsades de pointes. Magnesium prevents seizure in exlapmpsia.

    Mg2+ works by decreasing ACH in motor nerve terminals and acting on the myocardium by slowing the rate of SA node impulse formation an prolonging conduction time. Mg2+ appears to inhibit calcium uptake into smooth muscle cells, reducing uterine contractilty . in general Mg2+ relaxes excitable neural and muscular tissue

    Magnesium sulfate can lead to muscular weakness and loss of reflexes it causes diarrhea by promoting bowel evacuation through osmotic retention of fluid, which distends the colon. Severe magnesium toxicity can lead to respiratory paralysis
    __________________________________

    31 year old woman comes to the ER dept with palpitations, the EKG reveals a short PR interval, supraventricular teachycardia develops. When diltiazem is adminstered, the pt develops ventricular tachycardia. The pt remains hemodynamically stable .

    Wolff=parkinson white syndrome
    wolffparkinsonwhite 1 - The Medical student Review
    is best treated with procainamide or amiodarone. if an acute arrhythmia such as SVT or VT develops. these agents are effective against both atrial and ventricular arrhythmias, making them drugs of choice.

    Cardioversion for a pt hemodynamically unstable. this is defined as chest pain, SOB from CHF, confusion, hypotension defined as systolic BP <90

    radiofrequency catherter ablation after electrophysiologic studies is curative. The accessory conduction pathway is destroyed in an electrophysiology laboratory.

    _________________________________________

    Propylthiouracil (PTU) and Methimazole

    PTU and methimazole are the best initial therapies for hyperthyroidism. They are used to lower the level of thyroid hormone.
    they don't block the target organ effect. PTU is safe during pregnancy.

    PTU and methimazole inhibit the peroxidase enzyme. This blocks the oxidation of iodine, inhibits the incorporation of iodine onto tyrosine, and stops coupling of these compounds into T3 and T4. hence these meds block three essential steps in the formation of thyroid hormone.

    the most important adverse effect of these compounds is neutropenia. Both lower granulocyte levels by inhibiting bone marrow. Purpuric skin lesions may also occur.

    ______________________________

    50 yea old man with psostate ca comes to ER after having experienced lethargy, constipation and generalized weakness for four days. His calcium is markedly elevated at 14.5 and the WKG shows a short QT, hydration with saline and loop diuretics have been given

    Bisphosphonates are the best initial therapy to lower calcium levels after fluids and diuretics have been initiated. Bisphosphonates (pamidronate, zolendronic acid, alendronate, ibandronate risendeonate) are also used for osteoporosis when the T score is 2.5 standard deviation beloe normal (t-scre-2.5)

    bisphosophantes work by inhibiting osteocalsts, prevnting bone resorption. they bind calcium hydroxyapatite in the bone and prevent its dissolution.

    oral admin can cause esophagitis and for this reason they should be taken sitting up while drinking lots of water. rarer adverse is hypocalcemia and osteomalacia . osteonecrosis of the jaw has been associated with the use of bisphosphontes.

    >>>>>>>>>>>>>>>>>>>>>>>>>>>>
    The Medical student Review

    Text without context is pretext
    If your opponent is of choleric temperament, seek to irritate him 44845203 1 - The Medical student Review


  11. #28
    جوري's Avatar Full Member
    brightness_1
    Soldier Through It!
    star_rate star_rate star_rate star_rate star_rate star_rate star_rate star_rate star_rate star_rate star_rate
    Join Date
    Jul 2006
    Location
    من ارض الكنانة
    Gender
    Female
    Religion
    Islam
    Posts
    27,759
    Threads
    1260
    Rep Power
    258
    Rep Ratio
    89
    Likes Ratio
    23

    Re: The Medical student Review

    thought I'd forget about this thread today.. it has been a bad day sob7an Allah..
    maybe tomorrow will be better insha'Allah

    on with pharm /endocrine
    ___________
    Calcitonin

    is indicated for the following conditions
    acute hypercalcemia
    paget's disease
    osteoporosis

    calcitonin works predominately by inhibiting osteoclast activity.
    calcitonin can cause flushing, rash, and constiption it rarely causes depression and bronchospasm. The most common adverse effects are rhinitis, and flulike sx.

    ________________________________

    ok we now move on to derm

    minocycline
    is the best oral antibiotic for severe acne, that is not controlled by topical antibiotics or topical vitamin A derivatives such as terintoin. Other indicaations for minocycline include the following
    nocardia
    actinmycycosis
    rosacea

    minocycline is a tertracycline antibiotic that inhibits production in bacteria

    minocycline can cause blusing discoloration of the skin but doesn't cause photosensitivity rash as doxycycline can. miocycline can bring on vertigo by causing vestibular dysfunction.

    ________________________________

    a woman comes in with mild acne, she has a few comdones with an occasional inflammed papule or pustule

    a man has failed initial theray for acne and has numerous papules and postules with mild scarring

    your pt is very distressed and depressed because of numerous laege cysts on the face and trunk he is severely scarred

    case one mild comodonal acne, is treated with topical benzoyl peroxide and a topical atibiotic such as erythromycin or clindamycin. Benzoyl peroxide is both antibacterial and comedolytic. topical antibiotics will eliminate the causative organism proionbacteium acnes from the comoedones.

    case two, severe acne resulting in scarring in addition to benzoyl peroxide. the patient should be treated with a topical vitamin A derivative (tretionin) an oral antibiotic such as minocycline should be used.

    case three is severe cystic acne. an oral antibiotic and oral vitamin A are needed, isotrentinoin will decrease sebum producton but is extremely teratogenic and can cause severe depression, dry skin and hyperlipidemia
    _______________________________________

    an adolescent boy comes to see you for rx for a long standing itchy eczematous rash on his face, hands, and feet in the flexural areas, the skin is lichenifies from scratching, he also has seasonal rhinitis and occasional utricaria

    Atopic dermatitis is treated with antihistamines such as fexofendaine, cetrizine or loratadine. hydroxyazine and diphenhydramine are more potent but much more sedating. Doxepine is a tricyclic with extensive antihistamine effects.

    Atopic dermatitis sx can be lessedned in the long terms by moisturizing skim, avoiding harsh sopas and treating skin infections. although topical steriods are effective to control symptoms acutely, in the long term they lead to skin atrophy. Topical calcineurin inhibitors such as tarcolimus and pimercrolimus can control atopic dermatitis in the long term without using steriods.

    Antihistamines, such as fexfenadine are also used in the following condiions

    Allergic/ seasonal rhinitis
    angioedema
    utricaria

    _______________________________

    a woman comes to the offic for pruritic, silvery, scaly, lesions of the knees, elbows, and hands, the lesions are on the extensor surfaces

    localized psoriasis is treated with topical steriods to prevent skin atrophy. treatment with calcipotriene a vitamine D analog should be used. topical tazarotene, a vitamin A analog is also used. Tacrolimus and pimercrolimus are also used

    widespread disease is treated with UV light. the most effective systemic therapy is methotrexate, but it also has the most adverse effects. Biological agents such as etanercerpt, alefacept and efalizumab are used as alternatives to methotrexate.

    ______________________________________

    diagnostics infectious dz

    VDRL/RPR

    serume measurement for these nontreponemal examinations are used to diagnose or screen for syphilis

    in the primary stage, you would expect the patient to present with a chancre and painless regional lymphadenopathy. in the secondary stage rash, lymphadenopathy, condylomata lata and alopecia can be expected.
    teritiary syphilis is charcterized by cardiac or neurological dz.

    false positives connective tossue disease, infectious mono, malaria, leprosy, IV drug use, hep C, infective endocarditis and pregnancy

    false negatives prozone phenomenon- very high antibody titers are prestnt

    when your clinical sspicion is high yet the nontreponemal tests are negative, you would want to do a darkfield examination in primary syphilis and FTA-ABS test for other stages..

    The RPR and VDRL decrease or disappear in response to RX, the FTA can remain positive life long..

    _____________________________________


    Tzanck smear

    are used to dx the skin manifestation of either herpes simplex of varicella zoster. You are looking for multinucleated giant cells. Tzanck smear is not necessary if the diagnosis is clinically clear from the presence of vesicles.

    the most accurate test for a herpetic infection of the skin or genitals is a viral culture.

    _____________________________

    micro9 1 - The Medical student Review

    6624 1 - The Medical student Review

    arthrocentesis is a needle aspirate of synovial fluid for microscopic analysis. The cell count is the most accurate ways of telling is there is an infection vs inflammation.

    counts from 0-2000 are normal
    from 2000-20,000 are from inflammatory disorders such as gout. counts above 50,000 are from infections, counts between 20,000, to 50,000 are intermediate

    look for a pt with new onset of joint that is warm to the touch, painful, and swollen from an effusion, and decreased with mobility. serum uric acid maybe elevates

    gout is negatively birefringent needle shaped crystals, these are monosodium urate crystals

    pseudogout is positively birefringent rhomboid shapes crystals

    ___________________

    aldolase

    is released from damages skeletal muscle and is elevated in inflammatory myopathies such as polymyositis and dermatomyosistis. Creatine and CPK are also elevated.

    look for a case of an older woman with difficult rising from a chair, there maybe a heliotrope rash and Gottron's papules. Aldolase or CPK is the answer to the question best initial diagnostic

    the most accurate tests are the electromyogram and the muscle biopsy. these confirm the diagnosis of polymyositis and dermatomyosisits when the alodolase is elevated.
    _______________________________________

    Allergy testing

    is a subdermal exposure to trace amounts of suspected allergens with subsequent visual inspection for signs of hypersensitivity.

    this is contraindicated when there is a high risk of anaphylaxis.

    beta blockers such as propanolol should be stopped prior to testing for allergens for which there might be severe reactions during desensitization. If anaphylaxix occurs, we want to use epinephrine to reverse it, and beta blockers will make the epi less effective..
    >>>>>>>>>>>>>>>>>>>
    The Medical student Review

    Text without context is pretext
    If your opponent is of choleric temperament, seek to irritate him 44845203 1 - The Medical student Review


  12. #29
    جوري's Avatar Full Member
    brightness_1
    Soldier Through It!
    star_rate star_rate star_rate star_rate star_rate star_rate star_rate star_rate star_rate star_rate star_rate
    Join Date
    Jul 2006
    Location
    من ارض الكنانة
    Gender
    Female
    Religion
    Islam
    Posts
    27,759
    Threads
    1260
    Rep Power
    258
    Rep Ratio
    89
    Likes Ratio
    23

    Re: The Medical student Review

    wwwislamicboardcom - The Medical student Review

    was out for Eid today.. insha'Allah will resume tomorrow, might add another sections of most likely diagnosis if my time permits insha'Allah...

    The Medical student Review

    Text without context is pretext
    If your opponent is of choleric temperament, seek to irritate him 44845203 1 - The Medical student Review


  13. Report bad ads?
  14. #30
    جوري's Avatar Full Member
    brightness_1
    Soldier Through It!
    star_rate star_rate star_rate star_rate star_rate star_rate star_rate star_rate star_rate star_rate star_rate
    Join Date
    Jul 2006
    Location
    من ارض الكنانة
    Gender
    Female
    Religion
    Islam
    Posts
    27,759
    Threads
    1260
    Rep Power
    258
    Rep Ratio
    89
    Likes Ratio
    23

    Re: The Medical student Review

    I am going to introduce a new most likely dx here, will do it at a rate of 15/5/5 to catch up with the other topics..

    starting with cardiology

    ________________________________

    1- Man comes into the Er with chest pain, that changes with respiration. the pain is sharp, and is worsened by inhalation. He is short of breath, as well but the symptoms are hard to assess because a deep breath causes pain, so he takes short fast shallow breaths...

    most likely dx
    a- te pain changes with bodily position. it is worse when lying flat, and better when sitting up.

    b-cough, fever, hemoptosis
    c- sudden onset of SOB with a normal lung exam
    d-hx of astham or COPD with sudden onset of SOB, decreased breath sounds on one side

    pericarditis is associated with pleuritic chest pain that also worsens with changed in bodily position. typically the pain of pericarditis is releverd when the person sits up, and stretch is relieved from the pericardium. only 30 % of pts have a pericardial friction rub. if it is present, it helps answer, the most likely diagnosis question. if the rubs are absent, this excludes nothing EKG shows PR depression and diffuse concave ST elevation.

    Pneumonia is associated with cough, sputum, and hemoptysis. fever is nonspecific, every cause of pleuritic chest pain is asspciated with fever.

    Pulomnary embolus presents with the sudden onset of shortness of breath and clear lungs on exam. There is no charcteristic physical findings of PE to allow you to resolve the dx in question.

    sudden onset plus normal lings = PE. pt maybe on a long trip

    pneumothorax, when large breath sounds decrease on one side. obstructive lung disease ca predispose to pneumothraces, particularly when there are blebs with COPD.

    __________________________________

    A man is brought to the ER dept after losing consciousness at home, he wakes after a few mins

    a- sudden loss of consciousness and rapidly regaining consciousness, he is fully intact when regaining consciousness.
    b- sudden loss but was disoriented for an hour or two on regaining consciousness
    c-gradual loss of consciousness with shaking, sweating, palpitatons, and nausea

    d- has a 10 point rise in pulse and a 20 point drop in systolic pressure when going from lying to upright posture.


    crdiac syncope such as an arrhythmia or obstructive cardiac lesions results in the sudden loss and regaining of consciousness. Ventrcular rhythm disturbance such as a ventricular tachy or fib will result in syncope

    seizures can result in sudden loss of consciousness but the regaining of alertness is slow because of being post-ictal results in a gradual regaining of consciousness described as post ictal state.

    metabolic probs such as hypoglycemia, hypoxia or drug intoxication lead to a gradual loss of consciousness. this is often accompanies by signs of autonomic hyperexcitability such as tachycardia, palpitations and diophoresis. you may see metabolic respiratory acidosis or alkalosis.

    orthostatic instability leads to syncope in association with a >30 pt drop in systolic blood pressure on changing position with a 10 point rise in pulse.

    ________________________________________


    a patient comes to the office with palpitations for the last several weeks. she denies chest pain or SOB, the sensation is like her heart will flutter away at her chest..

    1- the pulse is irregularly irregular

    2- she drinks lots of coffee and the EKG is normal

    3- she is losing weight and has diarrhea, her eyes are bulged forwars (exophthalmos)

    4- there is episodes of flushing and low BP

    A fib presents with palpitations and an irregularly irregular pulse. arial rhythm disturbances rarely result in syncope.

    caffeine can easily lead to the feeling of palpitations even with normal EKG

    hyperthyroidism results in weight loss, anxiety, tachycardia, diarrhea, and palpitations, about one third have ocular findings such as exopthalmus


    carcinoid syndrome leads to papitations from the oversecretion of the neurotramsitter seratonin, episodic flushing, diarrhea, and episodes of HTN are common. palpitations with HTN should make you think of pehochromocytoma

    ________________________________________________


    patient comes to ER with palpitations found to have an SVT after admin of filtiazem his rhythm deteriorates to Vtach

    Wolff-Parkinson-white syndrome can present with an Atrial arrhythmia, alternating with ventricular arrhythmia. The key to answering the question is worsening of the rhythm after giving a calcium channel blocker CCB such as dilitizem or verapamil. the rhythm may also worsen with dig. CCB and DIg block conduction through the normal AV nodal pathway and force conduction through the abnromal aberrant tract. resulting in a deterioration of the rhythm.

    EKG showing a short PR interval or delta wave. EKG is the best initial test.

    elecreophysiological studies are the most accurate for pre-excitation syndrome.

    ___________________________________


    a pt comes for toutine visit on PE he is found to have a pulse of 45

    1- he is asymptomatic, he runs five miles each day
    2-he has canon 'a' waves in the his neck. occasionally he is light headed.

    sinus brady is common finding in well trained athelets. You can't be sure if bradycardia originates at the sinus node without an EKG

    third degree complete heart block is associated with canon 'a' waves in the neck. it is often associated with symptomatic hypotension or syncope, and that is why a pacemaker is necessary. canon 'a' waves result from atrial systole against a closed tricuspid valve. the only condition to have bradycardia and conon 'a' waves is a complete heart block.

    ________________________________________


    62 year old man is the ICU after an MI he is no suddenly lightheaded and hypotensive

    1- there is a holosystolic murmur at the apex radiating to the axilla, the lungs are congested.

    2-oxygen saturation increases from 40% in the right atrium to 82% in the right ventricle

    3- he had an inferior wall infarction. he has tachycardia and clear lungs

    4-bradycardia and canon 'a' waves are present

    mitral valve rupture leads to acute pulmonary edema, the murmur of mitral regurgitation is holosystolic and radiates to the axilla

    valve rupture leads to a step up in oxygen saturation as you go from right atrium to the right ventricle, this is from left to right cardiac shunting

    right ventricular infarction accompanies 30-40% of inferior wall infarctions. This is because they are both supplied by the right coronary artery. The lungs are clear, patient may also have rupture of both left ventricular free wall, but outcome then is immeiate death.

    complete heart block leads to bradycardia. hypotension and canon 'a' waves.

    ____________________________________________

    a man is admitted for an MI of the anterior wall, he suddenly loses pulse

    1- most likely DX

    2- best initial dx test

    3- initial therapy

    sudden loss of pulse can be from asystole, Vfib, vtach or pulseless electrical activity

    EKG is the best initial DX

    in Asystole start with epinephrine and atropine

    Vfib and Vtach-unsyncrhonized cardioversion
    pulseless electrical activity, correct the underlying cause such as tension pneumothoraz, PE, hypovolemia and tamponade.

    ____________________________________________

    28 year old woman seen for severe HTN, pressure is repeatedly elevated
    1- abnromal sound auscultated in the flanks of the abdomen
    2-hypokalemia
    3-episodic with palpitation
    4-upper extremity blood pressure is greater than lower extremity pressure
    5-hirsuitism, clitromegaly


    1- enal artery stenosis is most likely when bruits are heard on examination and most common cause of secondary hypotension
    2- when you see hypotension combined with hypokalemia, think Conn's sndrome, or primary hyperaldosteronism.
    3-pheochromocytoma is the only for of htn that is episodic
    4-coarctation of the aorta results in higher blood pressure in the arms compared to the legs. the pressure can also differ between the arms, if coarctation occurs before 'offshoot' of left subclavian artery.
    5-CAH with 11-hydorxylase deficiency leads to HTN when pt is under 30 and the HTN is very hard to control such as needing more than two antihypertensive meds.

    ________________________________________________

    Man presents wih SOB and TIA, he has intermittent fever. The murmur changes markedly with bodily posotion. The SED rate is high

    1-most likely DX
    2-Most accurate diagnostic
    3-RX


    atrial myxoma is a benign cardiac tumor that is charcterized by a murmur that changes with bodily positions. This is also called a tumor plop. Myxoma presents with a murmu that is similar to mitral stenosis because it obstructs diastolic filling. There are also systemic sx such as fever, elevated SED rate, and anemia. TIA is likely due to embolization of myxoma, myxoma can be friable

    Echo dignoses myxoma

    treated with surgical removal only.

    ____________________________________


    Man comes in with CP, Dyspnea and diaphoresis, occurs on exertion

    1- pain happens every time he walks one or two flights of stairs. EKG normal
    2-pain occured wih much less exertion. EKG shows ST depression
    3-EKH shows ST elevation

    stable angina is Cp occuring with the same level of exercise, stable angina is pain with exertion and relieved by rest, with a normal EKG

    unstable angina is a type of acute coronary syndrome with a worse pattern of chest pain or pain occurin at rest. Acute coronary syndrome is the proper name, because you can't tell if the cardiac enzymes such as the troponins will be elevated until later. this may turnout to be a nonstemi infarction if cardiac enzymes are elevated
    3- acute MI is assumes where there is chest pain with st elevation even before cardiac enzymes results are obtaines. The majority of pts with chest pain and stemi develop elevated troponins and CPK-MB levels. The pT would be a candidate for thrombolysis and or immediate cardiac cathe.

    _______________________________________________


    a man comes with pain his his leg, the last several weeks. the pain occurs while he is walking and is relieved when he sits down.

    1- pain is unilateral and occurs with any forms of exertion of the leg, the skin is mooth with hair loss and skin appendages.
    2- the pain is bilateral and made worse when walking downhill, he has no pain when bicycling.


    PAD occurs as pain with any form of exertion of the lower extremity and is relieved by rest. as it wosens, loss of skin appendages such as hair follicles and sweat glands. sx may improve when dangling the head of the bed, gravity increases blood flow to legs.

    spinal stenosis results in bilateral leg pain that is highly dependent on bodily postion. it is much worse with anything that has the pt leaning back. such as walking downhill. it is relieved by leaning forward such as sitting or bicyling. it isn't exertion that leads to pain, it is the pressure on the spinal cord on the ligamentum flavum in the spinal canal. MRI of the spine, likely lumbar will demonstrate stenosis.

    ___________________________

    34 year old woman comes to the office with palpitations and atypical CP. The pain has no fixed patter. PE reveals a mid-systolic murmur. The murmur worsens with valsalva and improves by a leg raise

    what is the dx and rx

    MPV is the most likely DX when the question describes atypical CP in a young female. There is a midsystolic click followed by a murmur. Valsalva will worsen only the murmus of MVP and hypertrophic obstructive cardiomyopathy

    MVP is confirmed with echo, and treated with beta blockers. Endocarditis prophylaxis prior to dental procedures is no longer recommended
    ________________________

    a healthy young man experiences SOB with exertion. he has an episode of syncope while playing basketball. exam reveals
    a systolic murmur worsens with valsalva and improves on squatting

    hypertrophic cardiomyopathy most often presents with SOB, it can also cause syncope and may lead to sudden death. The murmur has the same crescendo/ decrescendo pattern of aortic stenosis but is heard best at te lower left sternal border. Aortic stenosis is heard best at the 2nd right intercoastal space and radiates to the carotid arteries.

    HOCM shouls be treated with beta blockers. if Syncome occurs, an implantable cardioverter/defibrillator should be placed. Endocarditis prophylaxis prior to dental procedures is no longer recommended.

    __________________________________________

    woman comes with progressively worsened SOB on exertion and a murmur. there is edema

    1- pregnant woman with a diastolic extra sound followed by a murmur. she has dysphagia and hoarseness

    2-older man with angina and a systolic murmur radiating to the carotid arteries.

    3- diastolic decrescendo murmur with a wide pulse pressure

    mitral stenosis, often becomes symptomatic during pregnancy because of marked increase in plasma volume during pregnancy.

    dysphagia hoarseness happen from enlargement of the left atrium pressing on the esophagus and recurrent laryngeal nerve. another clue is opening snap

    aortic stenosis is a systolic murmur radiating to the carotid arteries and is the most common presentation of aortic stenosis

    aortic regurgitation presents with SOB, but this is a nonsepecif finding, the key to the answer is the diastolic decrescendo murmur at the left lower left sternal border and th wide pulse pressure.

    __________________________________________________

    A man with a hx of HTN comes to the ER dept with a sudeen onset of sharp CP radiating to his back, there is a 15 point difference in BP between the left and right arms. a dastolic decrescndo murmur is present .

    Aortic dissection presents with sudden onset of CP radiating to the bck, particularly the shoulder blades. HTN by far is the most common risk factor. the key to diagnostic is the pain radiating to the back, the wide pressure from aortic regurg and the difference in pressure between the arms.

    the best initial test is a CXRAY, which may show a widened mediastinum

    transesophageal echo, CT angiogram and MRI each have about a 90-95% sensitivity. aortic angiography is the single most accurate test. Transthoracic echo is not the test of choice due to its limited accuracy \; management is aggressive control of systolic BP (100-120mmhg)

    __________________________________________________

    ok.. now one with pharm endocrine
    Sitagliptin, exenatide, and parmlintide

    sitgaliptin, exenetide and pramlinitide are used as adjunctive therapies for diabetes. in addition to sulfonylurea, metformin or both. prmlinitide can also be combined with insulin.

    mechanism

    exenatide is an incretin mimetic. Incretin is released by the intestinal tract and increases after you eat. incretin increases insulin synthesis and release by pancreatic beta cells. Incretin decreases glucagon production

    sitagliptin- blocks degradation of incretin
    pramlintide- suppresses glucagon release via undetermined mechanism , delays gastric emptying, and has CNSS mediated anorectic effects

    most common adverse effect is hypoglycemia.

    ________________________________________
    Orlistat

    used to RX obesity
    is a reversible inhibitor of pancreatic lipases. This action blocks the absorption of fat in the intestine. fecal excretion is increases, this may cause mild decrease in fat soluble vits such as ADEK
    advers, oily stools, diarrhea, flatus, abdominal pain and other sx, similar to celiac d and steatorrhea.

    ___________________________________________

    fludrcortisone

    is a steroid with very potent mineralocorticoid activity. being adminstered, is is the closest thing to eating aldosterone.

    fludrcortisone causes sodium rention. it also causes hydrogen ion, acid excretion, leading to metabolic alkalosis

    fludrocortisone is the rx for the following conditions

    Addison's dz. (adrenal insufficiency)
    orthostatic hypotension
    septic shock rx (adjunct to hydrocortisone)
    type IV renal tubular acidosis

    fludrocortisone can lead to hypokalemia, edema, and HTN. like all steroids, fludcortisone can lead to osteoporosis, impaired wound healing and easy bruising
    _______________________________________________

    Alpha glucosidase inhibitors

    acarbose
    miglitol

    these are oral hypoglycemic agents that are used in the RX of Dm
    alpha glucosidase inhibitors are an adjunct for DM when sulfonylurea, metformin, or both are not able to control pts glucose level/ the pill must be taken with each meal to prevent glucose absorption

    the adverse are bloating, diarrhea and flatulance.
    ________________________________________________

    Metformin

    is a biguanide used in rx of DM
    it decreases hepatic gluconeogensis, and increases peripheral sensitivity to insulin. it doesnt require beta cells and will not therefore cause hypoglycemia.
    metformin may be associated with lactic acidosis. Metformin is renally cleared and therefore a rise in creatining can cause the levels to rise, metformin is contraindicated in RF

    metformin is the initial therapy for type II Dm, this is particularly true for obese pts who have peripheral insulin resistance.
    ______________________________________

    now diagnostics still in rheumatology

    ANA, double stranded DNA, and anti-SM antibodies

    ANA is more than 98% sensitive for SLE but ANA isn't specific for SLE.
    a negative ANA excludes SLE with 98% negative predictive value. Anti- DS-DNA antibodies are 99% specific for SLE. anti-smith antibodies are extremely specific for lupus.

    Answer ANA, anti-SM, and ds-DNA when you see a young female with multiple spontaneous abortions (lupus anticoagulant) rash, joint pain
    __________________________________________________ __

    Anticentromere antibodies

    Anticentromere are specific for CREST syndrome.
    CREST is calcinosis, Rynaud's phenomenon, esophageal dysmotility, telangeictasia, Antitopoisomerase (SCL-70) is specific for systemic sclerosis)

    anticentromere is the best initial test in a female 30-50 with positive rheumatoid factor positive DNA

    __________________________________________________ __

    Anti-histone antibodies
    are often associated with drug induced lupus SLE. The most common drug that cause lupus are procainamide, hydralazine, sulfonamides, INH, and pencilamine.

    stop meds
    ____________________________________


    anti-Ro and anti-La antibodies are associated with Sjögren syndrome presents with a positive ANA, anti RO/Anti La and positive for rheumatoid factor.

    best used when you see a triad of kertoconjunctivitis (sicca syndrome) xerostomia and arthritis, or dry eyes, dry mouth and arthritis
    _________________________________________________

    Anti-scl 70 antibodies

    are directed against topoisomerase-1 and are ordered when systemic sclerosis, particularly the diffuse type is suspected.

    look for polyarthralgia; Raynaud's phenomenon, thick, hidebound skin, dysphagia, and GI tract hypomotility as well as pulmonary and renal abnromalities.

    Anti-SCL 70 has a low sensitivity 30%. hence negative test will not rule out dz.

    it is however, strongly correlated with severe lung involvement, such as pulmonary fibrosis.

    >>>>>>>>>>>>>>>>>>>>>>>>>>>>>>>>>>>>>>>>>>>>>>>>>> >>
    Last edited by جوري; 10-02-2008 at 02:46 AM.
    The Medical student Review

    Text without context is pretext
    If your opponent is of choleric temperament, seek to irritate him 44845203 1 - The Medical student Review


  15. #31
    جوري's Avatar Full Member
    brightness_1
    Soldier Through It!
    star_rate star_rate star_rate star_rate star_rate star_rate star_rate star_rate star_rate star_rate star_rate
    Join Date
    Jul 2006
    Location
    من ارض الكنانة
    Gender
    Female
    Religion
    Islam
    Posts
    27,759
    Threads
    1260
    Rep Power
    258
    Rep Ratio
    89
    Likes Ratio
    23

    Re: The Medical student Review


    1- oh I know.. but when you are doing medicine you get no time off whatsoever
    2-You can't possibly remember everything, you need to always refresh. if you know the basic pathophysiology of things that you can apply it to any disease state, because the body only responds in a a certain number of predictable ways to any stressor.. but there is so much going on that you have to pay attention too, far to expansive for me to sum up in a couple of words here.
    3-I get my info mostly from books, but I am subscribed to two journals at this time. It is important to keep on top of them, because you'll find yourself answering incorrectly. for instance as a quick example. prophylaxis for endocarditis was mandatory for many a conditions, one which was valvular heart disease, the type of prophylaxis itself also had conditions, now you'll find the guidelines have changed. Also many meds go obsolete, some new ones are in.. you have to do continuing eduction yearly, plus need to always renew your ACLS every couple of yrs
    4- Do I enjoy it? well long gone the time when it was fun.. after a while it is just a very stressful life style and job, and alot of debt, loss of social skills, alienation from most family and freinds, lack of sleep, and inability to relate to others outside the confines of what you are taught, and how your own brain functions. basically life passes you by while you watch it.. but it has its rewards.. (I am yet to find them out) so don't let me discourage you if this is something you really want to do.. go in with a level head.

    for me I had to fight with my family to let me do this, my dad always thought women should be pharmacists, and in fact that is what my sis is, she has a doctorate in pharmacology (but she is miserable) I couldn't forsee myself in pharmacy (though it is in fact an excellent career for women. the pay is good and you can always work on a part time basis), I also had to fight with my own fears. and I had to balance the two, because I knew they were all waiting for me to fail to come out with the I told you so. I'd advise you get married before hand with someone going in the same field, because I don't think people outside of medicine are compatible for many reasons.
    1- if you are very stressed for any number of reasons, someone is suing you, the hours are bad, your liability insurance is up, your boss is an SOB and you come home to a nagging husband or a nagging wife, you are already in very bad shape.
    2-most of the time you want someone who can listen to you who actually appreciates what you mean.. you can't discuss how you've erroneously misdiagnosed someone in aortic dissection with a myocardial infarction thereby leading to their immediate demise, with someone who hasn't the faintest clue what you are talking about.
    They will judge you harshly, and unsympathetically on top of your own guilt and a heap of legal troubles you are already in.
    3-sometimes you want to come home and say nothing at all, and would also appreciate someone who can appreciate how silence is heavenly, or someone willing to take you on a car ride at 3 am simply because you want to have a good cry away from people's prying eyes and not have it be thought of a completely outlandish request.
    4- as for having kids.. I really can't comment on that any more than I can on a
    'good' marriage. It is something you'll have to work out with your uber understanding husband if such a man exists, hence I stress finding someone in your exact field.
    I'd advise in the end, that you go into a woman friendly field, and by that I promise you I don't mean Ob/gyne.. I mean something like dermatology, radiology, opthalmology/ pathology maybe peds.. all with the exception of peds are extremly competitive to get into.. you'll find 400 applicants competing for three spots (annually)

    so my advise, is genuinely have an earnest session with yourself and see if this is right for you. And if it is, then don't let anyone or anything deter you..
    in the end, I believe it to be the most noble profession and the highest level of education anyone can have, if those two are the only satsifactions one can get out of it..

    Allah knows best

    Last edited by جوري; 10-02-2008 at 08:25 PM.
    The Medical student Review

    Text without context is pretext
    If your opponent is of choleric temperament, seek to irritate him 44845203 1 - The Medical student Review


  16. #32
    جوري's Avatar Full Member
    brightness_1
    Soldier Through It!
    star_rate star_rate star_rate star_rate star_rate star_rate star_rate star_rate star_rate star_rate star_rate
    Join Date
    Jul 2006
    Location
    من ارض الكنانة
    Gender
    Female
    Religion
    Islam
    Posts
    27,759
    Threads
    1260
    Rep Power
    258
    Rep Ratio
    89
    Likes Ratio
    23

    Re: The Medical student Review

    I am doing this a bit early today.. 15/5/5

    questions ending cardio and moving on to others

    ________________________________________________

    a patient comes in with SOB on exertion, ortopena, bilateral lower extremity edema, and JVD. There is marked improvement with furosamide

    what is the DX
    1-multiple infarctions, alcoholism, and low ejection fraction on echo
    2-long HX of HTN and ejection fraction of 70
    3-hemochromatosis, sarcoid, amyloidosis hx.


    1-dilated cardiomyopathy presents with a HX of multiple infarctions or alcholism. The low ejection fraction and systolic dysfunction are key to the diagnosis. All forms of cardiomyopathy lead to SOB, dyspnea on exertion, edema, and orthopnea.

    2-Hypertrophic cardiomyopathy retains normal or hyperdynamic ejection fraction. Long standing HTN leads to impaired systolic dysfunction.

    3-restrictive cardiomyopathy is by far the least common cause of CHF, sarcoidosis, amylodosis and hemochromatosis are the key diagnosis...

    ________________________________________________

    A patient comes with a long HX of SOB, edema, Ascites, and hepatosplenomegaly. The pt. is an Immigrant. There is a rise in JVD distention with inhalation. There is a third heart sound

    1- what is the most likely diagnosis
    2-what is the most accurate diagnosis
    3-Most effective RX

    1- constrictive pericarditis presents with SOB and edema. This is nonspecific. The key to answering the diagnostic question is the presence of Kussmaul's sign, which is a rise in Jugulovenous pressure on inhalation. Constrictive pericarditis is most often a form of TB. which causes chronic inflammation of the pericardium. The third heart sound is the pericardial 'Knock'
    2- Most accurate test is a CT or MRI scan of the chest.
    3-the only effective therapy for constrictive pericarditis is surgical removal of the pericardium. The presenting sx of constrictive pericarditis can mimic those of restrictive cardiomyopathy. One of the distinguishing characteristics is the presence of equalization of the left and right sides heart pressure in constrictive pericarditis.

    __________________________________________________ ______

    moving on to ER medicine

    pt brought in after having lost consciousness and is now awake

    1- he has no focal neurological deficits and the CT scan is normal.
    2- he has weakness of the arm and CT scan shows a collection of blood and one dilated pupil
    3- his CT scan shows ecchymosis

    1- concussion is caused by head trauma. There can be loss of consciousness or altered mental status. There is no anatomic damage to the brain.
    The CT scan is normal.
    2-Subdural and epidural hematoma both lead to a collection of blood around the brain, visible on head CT. There can be focal neurological defects and a dilated pupil on one side.

    3-Ecchymosis of the brain results from head trauma. This is also called a 'contusion' Most of the time there are no focal neurological deficits. No surgery is required. Blood mixed in with brain. an ecchymosis is visible on brain CT and is, essentially, a bruise. No surgery is needed.

    ______________________________________________

    A patient has a sudden onset of high fever and a change in mental status. All cultures are negative and there is no neck stiffness. His CPK is elevated.

    1- he has recently been started on risperidone in addition to haloperidol
    2-he has just undergone major surgery
    3-He is outside playing volleyball in the summer

    1-Neuroleptic malignant syndrome presents with a high em and altered mental status in relation to starting neuroleptic medications. This is probably related to the antidopaminergic effects of these meds. CPK and potassium elevation can occur. treatment is with dantrolene and dopamine agonist medications such as bromocriptine.

    2-Neuroleptic melignant syndrome is caused by general anesthetics or succinylcholine. CPK elevations can also occur. Treatment is with dantrolene
    3- Heat stroke occurs in relation to dehydration and increased ambiebt temp. There is high fever and confusion. RX is with hydration, and physical measures to cool the pt. Aersolizing water and evaporation is the most precise method of cooling the patient.

    ____________________________________________

    Endocrinology

    a female child is brought in because of abnormal hair growth, she has not developed menstruation and has acne, hirsutism of her face, and abnromal balding.

    1-Hypotension, hyperkalemia, hyponatremia and elevated levels of 17-hyroxyprogesterone with diminished 11-deoxycortisone
    2- hypertension, hypokalemia and metabolic alkalosis. levels of 11-deoxycortisone are elevated

    1- 21-hydroxylase deficiency presents with hypotension, hyperkalemia and metabolic acidosis because of the loss of sufficient mineralocorticoid activity. Both aldosterone and 11-deoxycortisone levels decreased. Adrenal hormones are shunted into the excess production of DHEA, which accounts for ambiguous genitalia in females, such as clitromegaly. in addition, there is acne and hirsuitims. All forms of CAH have elevated ACTH and low levels of cortisol. 17- hydroxyprogesterone levels are increased because this is the precursor that should be converted by 21 hydroxylase.


    11-hydroxylase deficiency presents with HTN and hypokalmia so exact opposite of 21-hydroxylase deficiency.. because of increased level of 11-deoxycortisone (11DOC). 11 DOC has mineralcorticoid activity, which accounts for the HTN and metabolic alkalosis . Adrenal hormones end up shunted into the production of adrenal andrgens such as DHEA, Conn's syndrome would present similar but with elevated levels of aldosterone and decreased Renin.

    __________________________________________________ _______

    A man comes to ER with weakness, and orthostatic hypotension, he has hyperpigmented skin lesions, hyponatremia, hyperkalemia and metabolic acidosis. Dark lines visible on gums and teeth.

    1-Most likely DX
    2-Next step in management
    3-most accurate diagnostic test

    Addison's disease. and addisonian crisis are the loss of alodosterone from the adrenal gland resulting in loss of sodium and water and the development of hypotension, in addition, hypoaldosteronism results in hyponatremia, hyperkalemia and metabolic acidosis.
    Hyperpigmentation results from the high ACTH levels and high propiomelamocortin. Which also gives the dark lines in the gums.

    2- the most urgent step is to draw a cortisol level and administer saline and hydrocortisone! A specific mineralocorticoid such as fludrocortisone is often necessary. Acute treatment is more important than waiting for the results of specific endocrine diagnostic tests because of the risk of death from hemodynamic compromise.

    3- Cosyntropin stimulation testing is the most specific test. This is the measurement of cortisol levels before administering artificial ACTH. In a normal pt. the cortisol level witll rise with cosyntropin

    ____________________________________________

    a young man being evaluated for HTN. he has episodes of headaches, palpitations, tachycardia, and sweating along with HTN

    1-DX
    2-Diagnostic testing

    pheochromocytoma presents with episodes of HTN, palpitations, tachycardia and headache. The clue to the dx is the episodic nature of the HTN. the other sx are nonspecific.
    The best initial test is to obtain blood levels of free metanephrines. This is the more sensitive than levels of epinephrine and norepinephrine because the catecholamines are secreted in an episodic fashion and have a short half life. a 24 hour urine for catecholamines and metanephrines is highly sensitive and specific. as well a CT scan or MRI scanning of the adrenal glands if the catecholamine levels are elevated in order to localize the tumor.

    __________________________________________________ _____

    a patient comes in with muscular weakness, polyuria, and polydipsia. There is a metabolic alkalosis and the potassium level is profoundly low at 2.5 mEq/L

    1-HTN with low renin activity without edema.
    2-High renin and high aldosterone activity with an elevated level of urinary sodium until the body is depleted of sodium. urinary ca2+ is high. Normal BP.
    3- pt has a box of licorice in his hands. The renin level is low. BP is high.

    primary hyperaldosteronism, or Conn's syndrome, presents with HTN, hypokalemia and metabolic alkalosis. The plasma renin activity is suppressed because of HTN. high aldosterone levels with low renin is a hallmark of primary hyperaldosteronism. The pts muscular weakness is from low potassium. The polyuria is nephrogenic diabetes insipidus from hypokalemia.

    2- Bartter's syndrome
    4490381f1 - The Medical student Review
    Both patients have the typical facial appearance with prominent forehead, triangular face, drooping mouth, and large eyes and pinnae.

    this is a form of genetic defect in the loop of henle. pts lose sodium, chloride, and ca2+ resulting in volume depletion and secondary elevation of renin and aldosterone levels, with normal or low BP.

    3- Licorice contains a substance that is similar in function to aldosterone. Locorice ingestion will present in an identical fashion to primary hyperaldosteronism. Anything that gives a low potassium leads to muscular weakness. BP is high.

    ________________________________________________

    A man comes in with a long history of episodic flushing of his head and neck. the flushing is associated with strong emotions and the use of alcohol. He is hypotensive and tahycardic with the episodes. He has abdominal cramping and diarrhea, on PE, there are telangiectasia and the murmurs of tricuspid insufficiency and pulmonic stenosis.

    1-DX
    2-Best diagnostic

    carcinoid syndrome most often presents with episodes of cuteanous flushing in association with diarrhea nd tachycardia and abdominal cramping. Hypotension and tachycardia occur with the episodes. The recurrent episodes of flushing lead to vascular telengiectasia, Long standing disease is associated with right sided cardiac lesions from chronic exposure to sertonin. some pts have wheezing.

    2- best initial is urinary hydroxyindoleacetic acid (5-HIAA) levels. The tumors are localized in the intestinal tract with abdominal computed tomography (CT) and pentetreotide imaging (indium 111 octreotide imaging)
    metastatic - The Medical student Review

    ______________________________________

    a woman comes in for an offensive body odor and excess sweating, but she is unable to explain or resolve it.
    Her ring, hat, and shoe sizes have been increasing in the last few yrs. Her voice is thick and jaw protruding and enlarged. she also has joint pain

    1-what is the most likely DX
    2-Best initial test
    3-Most common cause of death

    Acromegaly is most often from a pituitary tumor secreting growth hormone. This leads to enlargement of hat, show, ring and glove sizes. The pt is in 30's and 40's. Arthropathy occurs from excessive articular cartilage proliferation. Entrapment neuropathies such as carpal tunnel syndrome can also occur. Diabetes occurs in 10-20% of pts. Amenorrhea can result from excess secretion of prolactin.
    2- best initial test is a level of Insulin like growth factor-1 (IGF-1) this is confirmed by findings of of a failure or GH suppression by the infusion of glucose.
    3-The most common cause of death is from the effects of GH on the heart, and HTN. There is also an increased risk of colonic polyps and cancer as well.

    ___________________________________________

    a woman comes in because of infrequent periods, Her menstrual abnormalities have been going on for several months. on PE she has galactorrhea. Her urine HCG is normal

    1-Dx
    2-Diagnostics

    Galactorrhea is an abnormally increased flow of milk from the breasts, It is caused by hyerprolactinemia. Medications such as alpha-methyladopa, tricyclic antidepressants or phnothiazines and beta blockers can cause it. It can occur normally from pregnancy and the first step is always a pregnancy test. head trauma can rupture he pituitary stalk and remove the normally inhibitory dopamine that comes down from the hypothalmus. if these levels have been excluded, MRI of the brain may show a pituitary tumor, prolactin inhibits the release of LH, and inhibits menstruation.
    2- measure the prolactin level. If it is markedly elevated in the absence of pregnancy, then an MRI is the most accurate test to detect a pituitary lesion.

    ___________________________________________

    a patient comes in with anxiety, unexplained weight loss, diarrhea, tachycardia, and palpitations. PE shows, tremor, thin hair, moist skin. The thyroxine level (T4) is elevated

    1- exophthalmos, skin abnormalities above the knee, and proptosis.
    2-An elevated thyroid stimulating hormone (TSH) level
    3-A tender gland
    4-Normal appearing gland, low TSH, low radioactive iodine uptake.

    Graves disease is charcterized by ocular and skin findings. The radioactive iodine scan reveals hyperfunctioning gland. The TSH is low, treatment is with methimazole or propylthiouracil (PTU) followed by radioactive ablation and hormone replacement if hypothyroid.
    2- TSH producing pituitary tumors are the only form of hypothyroidism associated with an elevated level of TSH. Perform and MRI of the brain to confirm dx.
    3-subacute thyroiditis is associated with a tender gland. The TSH level will be suppressed and the radioactive iodine uptake (RAIU) will be diminished
    4-silent thyroiditis is associated with normal appearing gland, low TSH, and low RAIU. The gland is nontender.
    _____________________________________________

    A young woman comes in because of failure to undergo menarche. She has normal breast development, but a paucity of pubic hair, vagina ends in a blind pouch and cervix is absent.

    dx
    rx

    testicular feminization or complete androgen insensitivity often comes to light when there is failure to achieve menses. The pt appears female, normal breast development, but there is a marked diminishement in pubic and axillary hair. the vagina is short, and the cervix, uterus, and ovaries are absent. testicles can be found in the abdomen or labia
    surgical removal of the gonad with estrogen replacement and dilation of the vagina in the management. these pts are emotionally and socially and functionally female (think jaime lee curtis)

    __________________________________
    on to GI

    72 year old man comes to ER with a sudden onset of severe abdominal pain, he has a HX of severe mid abdominal pain. He has a HX of aortic stenosis, and a fib. he has been losing weight. his abdominal exam is relatively benign compared to his sx, his stool is heme positive,

    dx, accurate diagnostic, effective therapy


    mesenteric ischemia presents with severe abdominal pain that is far more intense than the relatively benign exam, It is mid-abdominal. it often occurs in association with valvular heart disease. CAD, and Afib. Mesenteric ischemia is often an acute embolic event to the mesenteric artery.

    mesenteric arteriography is the most accurate diagnostic test.
    treatment is by exploratory laprotomy for possible resection of the affected segment of bowel. pt with signs of peritonits should go directly for laparotomy. if infarction occurs death is highly probable.

    ______________________________________________

    a man is brought to the ER with multiple episodes of vomiting blood, he also has diarrhea and black stool

    1- bleeding preceded with severe violent retching
    2- mid epigastric pain relieved by food
    3- he is an alcoholic with low platelets ad spider angiomata. The volume of hematemesis is enormous

    1- Mallory weiss tears are non-transmural tears in the esophageal mucosa. This is preceded by repeated episodes of retching or vomiting for any reason. Any form of upper GI bleeding can result in melena f more than 100-200 ml of blood is lost.
    2-duodenal ulcer is the most common cause of upper GI bleeding
    duodenal ulcers present with epigastric pain. the pain can be relieved by food. Endoscopy is necessary for specific diagnosis.
    3-Esophageal varices leads to the highest mortality of any form of GI bleeding. The case will describe severe liver disease, severe cirrhosis is often associated by splenomegaly with splenic sequestration of platelets.

    _____________________________________________

    now pharm
    ______
    still on endocrine
    Sulfonylureas

    Glyburide
    Glipizide
    Gliclazide
    Glimpiride
    Sulfonylureas are sulfa derived oral hypoglycemic medications used for DM
    they lower blood glucose by simulating pancreatic beta cells to release insulin. They bind ATP-dependent potassium channels on the beta cell, which inhibits the outflux of potassium. This depolarization opens voltage gated ca2+ channels leading to increased fusion of insulin with cell membrane. unlike metformin sulfonylureas require functioning beta cells.

    all sulfonylureas can cause hypoglycemia and weight gain. The increased insulin release prompts adipocytes to pick up more glucose. They are teratogenic and can cause SIADH . They are sulfa derivatives and are contraindicated in sulfa allergies.

    Sulfonylureas are first line oral hypoglycemics. metformin us preferred in obese pts.

    __________________________________________________ _
    1 63 year old woman with DM is seen for routine eval. Her urine shows microalbuminuria. Her dilated nose exam shows background retinopathy as well as some neovascurization

    what will protect her kidney, and eyes

    microalbuminurea in a diabetic should be treated with an ACE inhibitor
    Diabetic retinopathy can be controlled with tight control of her glucose level. Laser photocoagulation is indicated for proliferative retinopathy. defined as the presence of neovascularization of vitreal hemorrhages. Diabetic retinopathy is not controlled with or lipid control. Asa will not control diabetic retinopathy

    ________________________________

    a man with diabetes has glucose levels > 200 md/dl despite the use of maximal doses of several oral hypoglycemics agents. His Hemoglobin A1c is >9 %. in addition, his glucose levels fluctuate during the day

    Insulin glargine should be given as a once a day injection combined with one of the rapidly acting insulins (glulisine, aspart, or lispro) with meals

    insulin glargine rapidly reaches a peak level and maintains a constant level for 24hrs. Glargine is similar in effect to an insulin infusion pump. the short acting insuling (glulisine, aspart and lispro reach a peak effect in 50-60 mins and last for four hours, they seem to be equal in efficacy.

    ______________________________________

    ok now in GI pharm

    Interferon
    interferon alpha is used to rx the following
    chronic hep C in combo with ribavirin
    chronic hep c
    melanoma
    cryolgobilinemia in combo with ribavirin
    multiple sclerosis
    the mechanism of interferons are not clear, they are a cytokine that assists the immune response by inhibiting viral replication within the cells. they are immune modulatory and benefit multiple sclerosis
    interferons cause flu like sx, such as myalgia, arthralgia, depression, thrombocytopenia and leukopenia.
    _________________________________________________

    Ribavirin
    is a purine nucloside that is used as an antiviral
    rivavirin inhibits viral mRNA synthesis. it competitively inhibits cellular inosine 5'phosphate dehydrogense and interferes with the synthesis of guanosine triphosphate (GTP) and thus nucleic acid synthesis in general. it inhibits both RNA and DNA viruses.

    Ribavirin is the answer when the question describes a case of chronic hep c. the hepatitis C antibody should be positive with an elevated PCR RNA viral load and possible liver inflammation.. It is used in combo with interferon. Ribavirin is also the answer for respiratory syncytial virus.
    ribavirin can cause anemia

    _____________________________________

    diagnostics

    Rheumatology

    C-Anca (antineutrophil cytoplasmic antibody)

    is the best initial test to make a specific diagnosis of wegener's granulomatosis.
    C-ANCA is a blood test. A pattern is seen on indirect immunofluorescence microscopy that indicated antibodies directed against proteinase 3 of neutrophils.
    answer C-ANCA when you see upper and lower respiratory probs combined with renal dx, as hematuria and red cell casts in the urine. there will be hemoptysis, cough and sputum with abnormal chest xray not responsive to antibiotics. otitis and sinusitis will also be present.

    the most accurate test for wegner's granulmatosis is a biopsy of the lung and kidney
    _____________________________________________

    DEXA scanning
    is used to dx osteoporosis. it is used to detect bone density, when compared to general population
    this is used on post meopausal women and pts with high cortisol like cushing's syndrome, exogenous steroids, hyperthyroidism or with hyperparathyroidism.
    rx is with estrogens, bisphosphantes and SERM such as raloxifen and tamoxifen.

    __________________________________________________ __

    muscle biopsy
    is used to dx suspected myopathy such as
    glycogen storage disease
    3302650f3 1 - The Medical student Review
    vp - The Medical student Review
    gsdmain2 1 - The Medical student Review

    mitochondrial gene malformation
    13 CMTtb - The Medical student Review
    dystrophinopathies
    artsin190102tsaofig3 1 - The Medical student Review
    dermatomyositis
    2404 1 - The Medical student Review
    Gottron's papule
    and
    dermatomyositisheliotropeeyelidspicture 1 - The Medical student Review
    heliotrope eyelids

    and polymyositis
    13 polymyositis - The Medical student Review
    lymphocyte in myofibber
    .. this is done when you see proximal muscle weakness, weak shoulder abduction, weakness rising from a chair, this is usually associated with myalgia and muscle soreness. other signs of myopathy is recurrent myolysis.

    it isn't the answe for endocrine disoders like mysthenia.

    _________________________________

    RF factor
    present in 70% of pts with RA. RF isn't specific for RA
    .. do this test with a woman with symmetric inflammatory arthritis that s worse in the morning. affecting wrists MCP, PIP and look for elevated ESR or synovial fluid between 2000 and 20,000 cells.

    _____________________________


    cricket fig2 - The Medical student Review

    this is an upright xray of the chest. pt is sitting up so it can show air under the diaphragm

    perforated abdominal organ x ray, can be associated with penetrating trauma, rupture secondary to diverticulitis, appendicitis, other infection or iatrogenically from endoscopy
    >>>>>>>>>>>>>>>>>>>>>>>>>>>>>>>>>>>>>>>>>>>>>>
    Last edited by جوري; 10-02-2008 at 11:37 PM.
    The Medical student Review

    Text without context is pretext
    If your opponent is of choleric temperament, seek to irritate him 44845203 1 - The Medical student Review


  17. #33
    جوري's Avatar Full Member
    brightness_1
    Soldier Through It!
    star_rate star_rate star_rate star_rate star_rate star_rate star_rate star_rate star_rate star_rate star_rate
    Join Date
    Jul 2006
    Location
    من ارض الكنانة
    Gender
    Female
    Religion
    Islam
    Posts
    27,759
    Threads
    1260
    Rep Power
    258
    Rep Ratio
    89
    Likes Ratio
    23

    Re: The Medical student Review

    well, I had a bad day wal7mdlilah...
    did you have a bad day?

    will spare you nuances of mine if you spare me yours

    questions on with GI
    _____________
    Man brought into ER with multiple episodes of red blood in his stool

    Low GI bleeding is most commonly caused by diverticulosis and angiodysplasia. Other caused are polyps, colon ca. and ischemic colitis

    colonoscopy is the most accurate diagnostic test of lower GI bleeding. There is no definitive way to determine the precise etiology of colonic bleeding without endoscopy, barium studies, angiography, and CT scanning cannot lead to specific diagnosis.

    Hemorrhoids may also lead to red blood in stool. Often, the history will mention that the patient notes hematochezia and/or red blood on wiping.

    ___________________________________

    32 year old with one day of diarrhea, no blood per still
    1- there is vomiting, he recently ate chinese food
    2-he has recently been camping. He has bloating and flatulnce
    3-He is HIV positive with <CD4 cells.
    4-There is flushing and wheezing. He ate fresh fish the same day.

    1- Bacillus cereus, is associated with refried chinese rice. As with staphylococcus aureus there is no blood per stoll because it is a p preformed toxin. both organisms often present with vomiting

    2-Giardiaisis is associated with unfiltered water, such as found on camping trips. Bloating and flatus are common, Giardiasis can mimic fat malabsorption.
    3- Cryptosporidiosis is an organism that is common in those with AIDS and profound immunosuppression. The diarrhea is often chronic and responds to treatment of the underlying HIV disease.
    4-Scombroid is histamine fish poisoning. Bacteria that produce histamine infect tuna, Mackerel or mahi-mahi. resulting in the rapid onset of diarrhea, vomiting, flushing and wheezing.

    ____________________________________________

    35 year old woman comes to the office with several months of crampy lower abdominal pain. but there is never blood in the stool or weight loss.
    1- the diarrhea alternates with constipation. The pain is relieved with a bowel movement. Symptoms are less at night.
    2-she has episodes of flushing and hypotension
    3-dietary change relieves all the symptoms within 24 hours.

    1-Irritable bowel syndrome is a pain syndrome that is often associated with diarrhea alternating with constipation. sx are less at night. Key is abdominal pain with completely normal tests.
    2-carcinoid syndrome presents with episodes of flushing and diarrhea, hypotension. Urinary 5-HIAA confirms the diagnosis.
    3-Lactose intolerance presents with diarrhea in the absence of weight loss. Removal of milk products and cheese relieves the sx. Celiac disease would lead to weight loss that would need several weeks for sx to resolve. celiac disease would also be related to gluten containing products.
    _____________________________________________

    A generally healthy 40 year old teacher comes to the ER with several days of bloody diarrhea which occurred four times today. He has a temp of 102F pulse 105, and BP 112/78
    1-has has been eating raw oysters and clams
    2-he has mussels and has a hx of liver disease. Physical exam shows bullous skin lesions
    3-anemia and thrombocytopenia, and an elevated creatinine are present. The retic count, bilirubin and LDH are elevated, haptoglobin is low

    1- vibrio parahemolyticus is transmissited by shellfish. such as oysters, clams. Shellfish filter feeders that concentrate microorganisms as they feed themselves.
    2-Vibrio vulnificus is associated with diarrhea in pts with liver disease who consume contaminated shell fish. there is also incidence of developing bullous skin lesions.
    3-E.coli 0157:H7 is associated with the development of hemolytic uremic syndrome.

    ________________________________________________

    a patient comes in for epigastric discomfort that radiates up into the chest. He has cough and hoarseness and bad taste in his mouth, like he is sucking on pennies.

    Most likely DX?
    Most accurate test
    management?

    GERD leads to epigastric pain that radiates under the sternum. In addition, the acid hits the back of the tongue, leading to bitter taste in the mouth, when acid hits the vocal cords there is hoarseness and sometimes coughing and wheezing.

    the most accurate test is a 24hour PH monitor
    The first therapy is a proton pump inhibitor (PPI) is both diagnostic and therapeutic.

    ______________________________________________

    A man is evaluated for several weeks of epigastric discomfort and pain

    1- He is an alcoholic and there is epigastric tenderness
    2-he has no other sx. All lab tests are normal
    3-He has two episodes of black stool, the pain is better with food

    1-Pancreatitis is the only form of acute epigastric pains that is reliably associated with tenderness. Gastritis and ulcer disease are rarely associated with epigastric tenderness unless a perforation has occured
    2-Non-ulcer dysphagia is the most common cause of epigastric discomfort. There is epigastric pain with an entirely normal examination including a normal endoscopy and the etiology is unknown.
    3-Ulcer disease is the most common cause of upper GI bleding. Ulcers are not as common as non-ulcer dyspepsia as a cause of epigastric pain.
    duodenal ulcers are more often improved with eating. Gastric ulcers are worsened with eating. An upper EGD can be diagnostic.

    _____________________________________________

    a 22 year old woman comes to the office with recurrent episodes of diarrhea, fatigue, and abdominal pain. There is occasional blood in addition she has joint pain, erythema nodosum and uveitis. The hematocrit is 32, MCV is 90 and ESR is elevated. Alkaline phosphatase is elevated but the bilirubin is normal. Stool culture and ova/parasite exam show nothing.
    1-rectal bleeding is common. Antineutrophil cytoplasmic antibodies ANCA are present and antisaccharomyces cerevisiae (ASCA) are negative.
    2-perianal and small bowel disease is present. A fistula was present in the past. Granulomas are present on biopsy. ANCA is negative ASCA is positive. A mass is palpable in the abdomen.

    Ulcerative colitis with recurrent episodes of bloody diarrhea and pus from the rectum. the extra-intestinal manifestations of both forms of IBD are identical. Both give joint, skin, and ocular sx. Both give sclerosing cholangitis. UC gives positive ANCA and negative ASCA.

    Crohn's disease gives small bowel disease, fistulae, and perianal disease, in addition to 'skip' lesions. Granulomas are characteristic of CD. CD gives a negative ANCA and positive ASCA. Anemia, low albumin levels and a high sedimentation rate can be found in both disease. Crohn's transmural inflammation whereas UC is limited to mucosa.
    ________________________________________

    A woman is evaluated in the office for moderate hepatomegaly and elevation of the AST, ALT and bilirubin. A few spider nevi are present on skin.

    1- The antinuclear antibody (ANA) and anti-smooth muscle antibody are positive. Gammaglobulins are also elevated and there is a brisk response to prednisone
    2-Hepatomegaly is the main finding in a woman with diabetes, obesity and hypertriglyceridemia. the ALT is slightly higher than the AST. Fatty liver is seen on imaging. She doesn't drink alcohol.

    1-autoimmune hepatitis presents with hepatomegaly and the stigmata of chronic liver disease. The ANA is often positive and the gammaglobulin levels are elevated. less reliable findings are the presence of antismooth muscle antibodies and the liver-kidney microsomal antibody. Autoimmune hepatitis responds briskly to prednisone use.

    Non-alcoholic steatohepatitis (NASH) is associated with obesity, diabetes, hyperlipidemia. The liver biopsy shows fatty infiltration you would see in a pt with alcoholic use. NASH is associated with an ALT slightly greater than AST. this is opposite in a person with alcoholic liver disease. There is no definitive treatment for NASH besides losing weight and controlling the diabetes and hyerlipidemia .

    _______________________________

    a 38 year old man comes in with weight loss and flatulence, diarrhea, malodorous stool and weakness, he bruises easily, and his ca2+ is low. Sudan black stain is positive

    1- iron deficiency is present. Folate level is low. He has a skin rash with vesicles. Anti-gliadin and tissue transglutaminase antibodies are positive.
    2-Chronic alcoholic with epigastric pain and normal folate and iron levels.
    calcification of the pancreas on CT scanning. Lipase and amylase level are normal

    celiac disease and chronic pancreatitis both present with steatorrhea and weight loss. Both diseases lead to malabsorption of fat associated with the loss of ca2+ and vitamin K, easy bruising and malabsorption of vit B12. Only celiac disease leads to malabsorption of iron and folate. Iron and folate need an intact bowel to be absorbed. but don't need pancreatic enzymes to be absorbed. The most accurate test for celiac disease is small bowel biopsy ( do you see a theme here?.. I might be partial, but a pathologist is the arbiter of medicine) with them lie true diagnosis!

    Chronic pancreatitis is most often from alcohol. The iron and folate levels are normal. Lipase and amylase levels are normal in most patients with far advanced chronic pancreatitis. Calcifications are present on CT of the pancreas in 70-80% of pts . The most accurate diagnostic test is a secretin stimulation test. Secretin should provoke the release of bicarbonate -rich pancreatic enzymes in a normal person.

    _______________________________________________

    Man presents with dysphagia and weight loss
    1-pt is<50 with dyshagia for both solids and liquids at the same time
    2-65 year old man with a clong hx of ETOH and tobacco use. Dysphagia begins with solid and progresses with difficulty for liqueds.
    3-Foul breath and regurgitated food on the pillow the following morning
    4-HX of scleroderma with reflux sx
    5-CP that comes and goes. it is very severe but not associated with eating

    1- Achalasia is associated with dysphagia for both solids and liquids at the same time. it is not progressive. There is no association with smoking or drinking.
    2-Esophageal ca. is dysphgia first for solid food, then liquids. ca gets progressively worse, Achalasia is not.
    3-Zencker's diverticulitum is associated with foul smelling breath. Do not use nasogastric tube or endoscope because of the risk of perforation.
    4-Scleroderma esophgitis leads to reflux disease because the esophagus is ot capable of contracting. The answer is most likely DX question is easy scleroderma + esophagitis = scleroderma esophagitis. give proton pump inhibitors. look for CREST syndrome.
    5-spastic disorders of the esophagus present with pain not related to eating or exertion. to answer the question, it must include a negative EKG, and stress test so as to not be classified as angina. Esophageal manometry can be diagnostic.
    _______________________________

    A man comes in or evaluation of weight loss, diarrhea and easy bruising, the ca2+ level is low and the sudan black stain is positive.

    1- He has arthralgia, fever, and cognitive defects. There are ocular abnormalities such as nystagmus. Adenopathy is present. Biopsy of the duodenum shows PAS-positive organism
    2- a patient from the Caribbean has severe folate deficiency and VIt B12 deficiency. Biopsy shows abnromal villi with lymphocytic infiltration. Antigliadin and anti-endomysial antibodies are negative.

    Whipple's disease is a cause of malabsorption with arthraligias, fever and CNS abnormalities. Presence of PAS positive organisms, best initial therapy is a year of trimethoprim/sulfamethoxazole

    2- tropical sprue is the answer when malabsorption in association with severe folate and B12 malabsorption. The Q must give a hx of someone from the Caribbean or India. On biopsy the villi are abnromal with inflammatory cells but they are not as flat as those seen in celiac disease. treatment is with tetracycline and folate.
    __________________________________________________ _________

    An alcoholic man is admitted with severe epigastric pain, abdominal pain, nausea, and vomiting, he is restless with mild fever.
    1-Pt has an elevation of his amylase and lipase levels, as well as urinary trypsinogen activation peptide. CT scan shows inflammation.
    2-CT scan shows necrosis of >30%
    3-CT shows necrosis and biopsy grows gram negative organisms

    1-Acute pancreatitis occurs in alcoholics, and those with gallstones obstructing the ducts. Epigastric pain, nausea, and vomiting are present. the key dx is epigastric tenderness in an alcoholic. Trypsinogen activating peptide is elevated. treatment is pain control, IV fluids, and NPO until pain is resolved.
    2-Necrotizing pancreatitis on a CT scan of the abdomen is much more important as a prognostic factor than Ranson's criteria.. pts with severe necrosis should undergo biopsy to see is an infection is present. Necrotizing pancreatitis may benefit from antibiotics such as imipenem to prevent infection
    3-onfected necrotizing pancreatitis can be diagnosed only by biopsy or surgery. these pts have nearly 10% mortality without surgical debridement.

    __________________________________________________

    a woman comes in with severe itching, hepatomegaly and elevation of alk phos and GGTP.

    1-HX of IBD. overtime the bilirubin begins to elevate
    2-middle aged woman with xanthomas, fat soluble vitamin malabsorption, hyperlipidemia and skin hyperpigmentation.

    1-primary sclerosing cholangitis occurs in those with IBD. The alk phos. is elevated and the bilirubin only elevates much later in the disease. RX with ursodeoxycholic acid, but it is of limited effect
    2- primary billiary cirrhosis occurs in middle aged women who present with itching elevated alk phos. The most most accurate test is the anti-mitochondrial anti-vody. RX is with ursodeoxycholic acid, but it is of limited benefit.
    _____________________________
    a young man is referred by psych for eval of tremor, and choreiform movement disorder. He was admitted for paranoia and psychosis, but was found to have an elevation of his transaminases and a coombs-negative hemolytic anemia.
    dx, diagnostic test. therapy

    Wilson's disease is a form of copper deposition in the bran, liver and kidneys. In addition, there is coombs negative hemolytic anemia. look for liver disease with a movement disorder and psychosis.
    Wilson's disease is diagnosed by finding kayser fleisher rings on slit lamp exam. as well a low level of ceruloplasmin, which is the copper carrying protein in the body. There is increased urinary copper excretion, although the single most accurate test is an increased copper level on biopsy.
    penciliallmine is the treatment that removes copper from the body.
    __________________________________________________ ____

    Middle aged man comes in for evaluation of joint pains and fatigue. He has hepatomegaly on examination and skin hyperpigmentation. diabetes has developed over the past few months. He has lost libido and has developed ED. LFT's are elevated. Echo shows restrictive cardiomyopathy.
    dx.diagnostic. therapy

    hemochromatosis is a form of iron deposition in multiple organs in the body, especially the liver. Cirrhosis will develop if untreated in 60% of pts with hepatocellular ca. in 15-30%. Another 15% will die of cardiac involvement. iron depostion also leads to diabetes, pseudogout, skin hyperpigmentation (bronze diabetes) and erectile dysfunction. the latter is a form of iron deposition in the pituitary and the gonadotropins.
    the best initial tests is iron studies, with an elevated iron and ferritin level and low iron binding. This is a high iron saturation. This prompts the most accurate tests, which are the HFE gene mutation. liver biopsy with increased iron is the is single most accurate test.
    Phlebotomy is the most accurate way to remove iron from the body.

    _____________________ ok on with diagnostics

    ER medicine

    dclvv1 1 - The Medical student Review

    this is a skeletal survey
    used in suspicion of child abuse, multiple myeloma and metastatic bone tumor.

    parents might say he fell, but you see bruises.. also a clavicle is a hard bone to break unless by physical abuse..
    _____________________________

    r01nv24g3b - The Medical student Review

    voiding cystourethrography
    is a radiological visualization of bladder filling and emptying through the urethra. not to be confused with cystometrics, which is a pressure measurement of the bladder, useful in evaluating incontinence. cystourethrography is an evaluation of the anatomy of the urogenital system

    for trauma
    neoplasia in and surrounding the bladder
    UTI (in children)
    akk children younger than 5 yrs old with UTI
    children of any age with febrile UTI
    boys of any age with UTI
    __________________________________________________ ______

    diagnostic peritoneal lavage

    look for drunk or unconscious person who can't have his abdomen examined reliably with multiple trauma and unexplained shock. a catheter is inserted into the abdomen and saline infuses, aspirate is examined.
    the following aren't appropriate for a DPL.
    pts with signs of acute abdomen, an indication of a perforated viscus or intrabdominal bleed is an indication for immediate surgical exploration.
    pts with gunshot wounds to the abdomen. is indication for immediate surgical exploration
    hemodynamically stable pt with hx of abdominal trauma but no definite signs of viscus perforation on PE. abdominal CT is indicated here.

    to be considered DPL positive DPL should show RBC count >100,000/mm3, WBC>500/mm3, bile or feces.
    DPL is inadequare to dx retroperitoneal injuries .
    ______________________

    9481 - The Medical student Review
    intravenous pyelogram (IVP) is an old test of virtually no clinical utility. was used to dx pts with renal colic when no stones were found on radiograph.

    IVP has been replaced with a US as the best initial, and CT as most accurate. IVP uses contrast which is a risk for both renal failure and allergic rxn.
    ________________________

    pelvic laparoscopy
    direct visualization of pelvic structures, most accurate for

    ectopic pregnancy
    PID
    endometriosis
    pelvic cyst
    chronic pelvic pain.

    for pt who presents with pevic pain vaginal bleeding, positive beta hCG, particularly if beta hCG is low for gestational age.
    also in PID where pregnancy is excluded.
    ___________________________________
    Pharm on GI
    43 yo with chronic hep B in the clinic evaluated for rx. her surface antigen has remained (+) for more than 6m. her hep HBe antigen is (+) as well.

    chronic hep B is defines as persistence of surface antigen for longer than 6 month. Entecavir, lamivudine, telbivudine, adefovir and interferon are all approved for use. therapy is most effective for those who are (+) for hep B e antigen (HbeAg) or hepatitis B DNA polymerase.

    Enecavir, lamivudine, adefovir, and telbivudine all inhibit viral reverse transcriptase (DNA) polymerase. They cause DNA chain termination.

    Entecavir, lamivudine, adefovir, and telbivudine all cause lactic aciddiosis, and can cause acute hep exacerbation. Adefovir is neprotoxic. interferon causes flu like sx, myalgias, arthralgia and headache.

    ___________________________________________

    40 year old woman w persistent epigastric pain, has large gastric ulcer, there is no malignancy on biopsy. but (+) for H-pylori

    initial therapy for H pylori is PPI, such as lansoprazole, omeprazole, pantoprazole, rabeprazole or esmeprazole combined with two antibiotics, the preferred are clarithromycin and amoxicillin.

    if initial therapy fails, rx with PPI combined with bismuth subsalicylate and two different antibitics tetracycline and metronidazole.
    ___________________________________________

    56 year old man with acute variceal hemorrhage from alcoholic cirrhosis comes to the er, he has been started on IV fluids, he received a blood transfusion and ffp

    1-octreotide (somatostatin) is used for variceal bleeding to dec the severity of the hemorrhage
    octreotide is a synthetic somatostatin, it decreases portal pressure and splanchnic blood flow, because there are numerous somatostains receptors throughout the portal circulation. it is superior to vasopressin or sclerotherapy. Propranolol would decreases frequency of recurrent bleeding but not has no effect on acute bleeding.
    no sign adverse of octreotide
    if octreotide doesn't work, endoscopic band ligation is the next best step. transfuse as needed, however massive transfusion can cause coagulopathy.

    somatostatin is used to control growth hormone release in pts with acromegaly who are not candidates for surgery. it is also used to contrl dirrhea and carcinoid syndrome. it helps treat glucagonoma.

    _______________________________
    48 year old with altered MS secondary to hepatic insufficiency

    Lactulose is the most effective theray for hepatic enecphalopathy. it can also be used to RX constipation. it is superior to neomycin as the best initial therapy for hepatic encephalopathy
    lactulose is a nonabsorbed disaccharide. Bacteria is the colon consume the lactulose and lower the ph of the bowel. the acidification changes ammonia (NH3) to ammonium (NH4+) ammonium isn't easily absorbed and more readily excreted.
    lactulose causes bloating, diarrhea and flatulance. it can result in hypernatremia and hypokalemia
    __________________________________________________ ___

    pencillamine is a PCN derived chelating agent used for a number of toxicities

    pencillamine chelates, copper, mercury, zinc, and lead, it also decreases T cell activity and rheumatoid factor.

    it can cause nephrotic syndrome and bone marrow suppression.
    pencillamine is the drug of choice for wilson's disease, sometimes used for lead poisoning and severe RA. not responding to other therapies. It is used to rx cystunuria, can also be used to rx arsenic and mercury poisoning.
    >>>>>>>>>>>>>>>>>>>>>>>>>>>>>>>>>>>>>>>>>>>>>>>>>> >>>>
    Last edited by جوري; 10-04-2008 at 02:08 AM.
    The Medical student Review

    Text without context is pretext
    If your opponent is of choleric temperament, seek to irritate him 44845203 1 - The Medical student Review


  18. #34
    جوري's Avatar Full Member
    brightness_1
    Soldier Through It!
    star_rate star_rate star_rate star_rate star_rate star_rate star_rate star_rate star_rate star_rate star_rate
    Join Date
    Jul 2006
    Location
    من ارض الكنانة
    Gender
    Female
    Religion
    Islam
    Posts
    27,759
    Threads
    1260
    Rep Power
    258
    Rep Ratio
    89
    Likes Ratio
    23

    Re: The Medical student Review


    today it will be 10/5/5 in lieu of 15/5/5.. I find this a bit time consuming, more so than I'd like it to be, and I still think we can be up to speed at some point

    ok still on GI in most likely dx
    ________________________________________

    A man comes to the ER with abdominal pain, tenderness and fever..

    1- hx of alcoholic cirrhosis and ascites. BP and pulse are normal
    2-HX of peptic ulcer dz. He has a BP of 86/60 and a pulse of 120, and there is rebound tenderness on PE.

    1- spontaneous bacterial peritonitis (SBP) occurs with ascites. The Dx is based on ascitic fluid cell count of >250 neutrophils. Culture of the fluid should be injected into blood culture bottles. Most commonly SBP is from a single organism, such as Ecoli. The Ascitic fluid protein level is low. RX is with CEFOTAXIME. Ascitic fluid should be sent for gram stain, culture protein, albumin, CDH, amylase, and cell count
    2- secondary peritonitis occurs from perforation of a viscus, is associated with signs of severe sepsis such as hypotension and tachycardia. peritoneal signs such as rebound or guarding are common. The ascitic fluid protein is elevated. This form of peritonitis must be treated surgically in addition to antibiotics. Look for air and the diphragm on an upright cxray.

    __________________________________________________

    A man comes for evaluation of recurrent peptic ulcers. The ulcers are multiple, >2CM in size and located in the distal portion of the duodenum. RX of H pylori has resulted in no benefit. he also has diarrhea..
    DX, Diagnostic test, and best therapy


    Zollinger Ellison Syndrome (ZES) is the most likely DX when the question describes a pt with ulcers that are large, distal, multiple and recurrent after RX for H pylori. Most ulcers are , 1cm in size. Diarrhea is from the inactivation of lipase from the high acid level.
    Most accurate diagnostic test is an elevated gastrin level when off H2 blockers or PPI. Secretin should normally suppress gastrin. In ZES, secretin causes a rise in gastrin levels.
    local dz should be resected. Metastatic dz should be treated with life long PPI.
    __________________________________________________ _____

    An elderly man is brought to the ER dept with SX of tachycardia, diaphoresis and lightheadedness that begin 15-30 mins after eating, he had surgeries in the past for nonresolving ulcers. Another hour or two after eating the sx recur.
    Dx. therapy

    this is dumping syndrome, occurs in those with vagotomy and gastrectomy as a part of surgery for ulcers. There are two phases with similar sx. A rapid release of gastric contents into the duodenum, resulting in an osmotic draw of fluids into the intestine, that results in hypotension, lightheadedness, tachycardia, palpitations and sweating. Later there is a rapid release of insulin resulting in hypoglycemia, which produces many of the same sx.

    Dumping syndrome is managed with multiple small meals, devoid of carbs. Dumping syndrome is seen in those with morbid obesity that have undergone gastric bypass surgery.

    _______________________________________

    A pt with longstanding diabetes comes to the office for evaluation of nausea, vomiting, anorexia with a sense of early satiety, and abdominal bloating, sometimes there is diarrhea and sometimes constipation..
    most likely DX, most accurate DX, what is the therapy?

    diabetic gastroparesis is a form of autonomic neuropathy occurring in pts with longstanding diabetes and its effects on the nerves in the stomach. There is bloating with early satiety. The major stimulant to gastric motility is stretch. Longstanding diabetes results in a neuropathy that reduces he ability of the GI tract to perceive stretch. DX is made with nuclear gastric emptying study.
    promotility agents such as metoclopramide and erythromycin will relieve sx.
    ___________________________________

    an elderly woman comes in for her evaluation of urinary incontinence
    1- there is an irrepressible need to void. it often happens at night, she leaks urine before she is able to go to the bathroom.
    2-The pt is obese. Episodes of incontinence are brought on by laughing, sneezing, coughing or lifting heavy objects.

    1-urge incontinence presents with sudden and irrepressible urge to urinate that results in the passing of urine before the patient is able to make it to the bathroom. There is often associated pain over the bladder. The Most accurate test is urodynamic studies in which a catheter with a pressure transducer is placed in the bladder with the bladder half full to measure pressure. treatment with urge incontinence is with anticholinergic agents that are more specific to the bladder like oxybutynin, tolterodine, darifenacin, solifenacin and occasionally tricylic antidepressants..

    Stress incontinence presents with leakage of urine associated with coughing, laughing or sneezing, which increase intrabdominal pressure. RX is with kegel exercises or topical estrogens cream. Estrogen creams increases the growth of the distal third of the urethra.
    _______________________________

    most likely dx for
    1-pt with an elevated alk phos but no itching and a normal anti-mitochondrial antibody test.
    2-bowed legs worsened slowly over time with an abnromal gait. Back pain and an enlarged skull with headaches. The alk phos and urinary hydroxyproline levels are elevated.
    3-75 y/o woman has a pruritic eczematous rash over her nipple. It is progessive and now has a custing and a discharge that is sometimes bloody.

    1-Paget's disease of the bone is often asymptomatic with just a marked elevation of alk phos.. Abnromalities of the x-ray can be found when x-rays are done for other reasons. Asymptomatic paget's disease doesn't need therapy.
    2-paget's disease is the answer when there is bone pain, headache and physical enlargement of the head, and bowing of the tibia secondary to softness. pain is the first sx. When very severe, there is warmth palpated over the bone. rarely extra bone growth is so severe that high output CHF develops. the alk phos is markedly elevated with normal ca2+ and phosphate levels. X-rays is the best initial test. The best initial therapy is bisphosphonates or calcitonin.
    3- paget's disease of the breast is a form of breast cancer in older women presenting with a prurituc, eczematous rash that sometimes develops a discharge. biopsy is diagnostic and rx is surgical.
    ____________________________________________

    A generally healthy 40 year old man comes in with severe pain in the bottom of his foot, the pain is extremely severe as he gets out of bed. in the morning and it improved with the first few steps. Stretching improves the pain. there is marked tenderness at the midpoint of the heel.

    dx, rx?

    plantar fasciitis is an idiopathic disorder of severe pain in the bottom of the foot. The pain is extremely severe in the morning, especially with the first few steps. There is severe point tenderness at the heel where the fascia inserts.

    plantar fasciitis improves gradually over time. stretching the foot with a towel or with a small wall stretch will improve condition. occasionally steroid injections or surgical release are necessary.

    _________________________________________________

    A young mother comes to see you because of pain in her wrist. the pain on the thumb (radial) side of the wrist occurs when she is gripping and object. the finkelstein test is abnormal

    dx, finkelstein? RX?

    De Quervain's tenosynovitis is pain in the tendons of the wrist. The etiology is unknown. DeQuervain's is the answer when there is pain, swelling, and tenderness on the radial side of the wrist.
    Finkelstein is pain in the wrist when the thumb is placed in the closed first and the hand is tilted toward the little finger (ulnar deviation)
    there is no proven therapy. NSAIDS and splinting are the mainstay therapy.

    ______________________________________

    a 54 year old woman comes to the clinic for a follow up because of fatigue, CBC reveals a decreased hct of 32% and an MCV is low as 68fl

    1- an elevated red cell distribution width RDW and a high PLT count
    2-a low serum iron, low TIBC, LOW retic, hx of RA
    3-A profoundly low MCV with very few sx, and an elevated red cell count, iron studies are normal


    The most common cause of microcytic anemia is iron deficiency. iron deficiency is associated with elevated RDW because the cells become progressively smaller as the iron deficiency worsens over time. Iron deficiency is associated with thrombocytosis. this is benign and requires no additional rx. the best initial test is a low iron, low ferritin and increase TIBC. hx of blood loss.
    2- anemia of chronic disease is characterized by a low serum iron, low TIBC, normal ferritin levels. Any chronic or inflammatory condition can lead to anemia of chronic dz. and is extremely common in RA
    3-Thalassemia is associated with very few sx because the red cell count is elevated. This can maintain the total HCT close to normal. Thal gives normal iron studies. the most accurate test is hemoglobin electrophoresis. Don't treat with iron.
    __________________________

    an alcoholic comes to the ER because of fatigue, his only med is isoniazid. stool negative for occult blood. Hematorcit is 32% and the iron is elevated.

    DX, diagnostic, therapy

    sideroblastic anemia is most commonly associated with alcoholism. and is the only anemia associated with a high circulating iron level.
    The MCV is most often increases, but it can be elevated or normal. Although leads poisoning is commonly associated with sideroblastic anemia, there are more people who drink alcohol than exposed to lead.
    Sideroblastic anemia is also associated with isonizid use and myelodysplasia.
    the most accurate test is a prussian blue stain. iron built up in ringed sideroblasts (see previous pages for smear). iron is built up in mitochondria.
    there is no specific therapy, either remove the toxic exposure or treat the myelodysplasia.

    ________________________________________

    ok pharm GI

    an alcoholic is admitted for an episode of vomiting blood from esophageal varices. he has has several episodes before and has been twice treated with esophageal band ligation. he is already being transfused and hydrates and has a normal PT time.

    acute esophageal bleeding can be treated with endoscopic band ligation. if this has been done, then transvenous( or transjugular) intrahepatic portosystemic shunts (TIPS) is useful. A catheter is placed into the hepatic vein, and a shunt is created through the liver to the portal veing. It can result in immediate decrease in portal HTN. the most common wrong answer is sclerotherapy, since it is inferior to band ligation.

    most common adverse effect of TIPS is hepatic enecphalopathy.
    propanolol is used to prevent recurrent variceal bleeding.
    ___________________________________________

    pt is very distressed by persistent sx of epigastric pain from GERD. still has SX in spite of PPis

    Nissen fundoplication is the treatment most likely to benefit a pt with persistent reflux GERD sx despite PPIs. this is surgically places narrowing of the distal esophagus in which the stomach is sewn around the esophagus.

    H2 blockers are never as effective as PPis.
    Cisapride is a promotility agent that has been removed from the market for causing arrhythmias. (caused torsades de pointes.
    pylor doesn't cause GERD, treating it won't help.
    distal esophagectomy is the rx for high grade dysplasia from Barrett's esophagus.
    ________________________________________________
    38 year old woman comes to the office with dysphagia with ingestion of both solis and liquids. she waked up with regurgitated food particles on her pillow. Barium studies show narrowing of the esophageal sphincter to relax

    Achalasia is treated with esophageal dilation. Pneumatic dilation is performed. In those refusing Pneumatic dilation. Botulinum toxin injection can be performed. the problem with botulinum toxin injection can be performed. The problem with Botulinum toxin injection is that it is not a permanent procedure and the symptoms can recur after 6-12 months, requiring additional injections or therapy.
    if pneumatic dilation or botulinum toxin injection repeatedly fail, surgical intervention is performed. The Heller myotomy is the surgical cutting of the lower esophgeal sphincter.
    _______________________________________________
    27 yr old healthy GI fellow has developed severe, bloody diarrhea and abdominal pain over the last day. He is having 10 bowel movements a day, his pulse is 125, temp 103F. He is orthostatic and has abdominal tenderness. he is vigorously hydrating himself.

    severe bloody diarrhea from food poisoning is treated with antibiotics, if the pt shows signs of sepsis. the question will describe hypotension, bloody diarrhea and abdominal pain.
    the best empiric therapy for severe infectious diarrhea is fluroquinolone such as ciprofloxacin. Cipro has the greatest range of coverage of invasive pathogens such as campylobacter and salmonella.
    __________________________________________________

    53 year old diabetic for chronic abdominal pain and bloating, and a sense of fullness, she also has nausea with occasional vomiting. she sometimes has constipation and sometimes diarrhea. Upper endoscopy is normal.
    diabetic gastroporesis is characterized by abdominal bloating, fullness, and early satiety and nausea, the best therapy for diabetic gastroparesis is with erythromycin or metoclopromide.

    Erythromycin increases the release of motilin in the gastrointestinal tract. This increases the force and frequency of contractions in the stomach and intestines. In a normal pt with diabetic gastroparesis, this increases the forward flow of gastric contents. Metoclopromode increases of gastric contractions as well.

    __________________________________________________ _____

    Laparoscopy1 - The Medical student Review
    diagnostic abdominal laparoscopy is used for direct visualization of abdominal structures used for
    Ascites of unclear etiology
    liver dz with inconclusive biopsy
    staging of abdominal ca .

    in trauma pt.
    maybe used in stable pts as a less invasive aalternative to a laparotomy. DAL is especially useful in investigating injuries of the diphragm.

    look for a victim of MVA in whom the CT is nondiagnostic and organ damage is suspected
    ________________________________________________
    prental screening

    tests include
    cervical cytology
    CBC, urinalsis, blood group Rh, serology for syphilis, rubella antibody, glucose screening, fetal ultrasouns, cervical cultures for Nisseria, group B strep, chlamydia trachomatis.
    in a woman over 35 chromosomal screening.
    _______________________________________________

    kidney ultrasound - The Medical student Review
    renal ultrasound
    best for
    kidney stones, Congenital renal malformations, pediatric tumors, recurrent UTI, chronic renal failure, renal artery stenosis, polycystic kidney disease

    in a pt with unexplained hemauria and flank pain and tenderness
    renal US in a pt with dysuric sx such as frequency and burning, the urinalysis has WBC and flank pain.
    _______________________________________
    v5c18d 1 - The Medical student Review

    right upper quadrant ultrasound
    used in pts with right upper quadrant pain with or without fever, the ultrasound will evaluate hepatobilliary structures for the folowing
    liver pathologies
    Masses, tumors, abscess, cysts
    portal HTN
    Gallbladder and billiary tree
    cholelithiasis
    cholecystitis
    choledocholithiasis
    cholangitis
    the most accurate test for billiary pathology is ERCP or MRCP. most accurate for liver pathology is liver biopsy.
    a (+) murphy's sign is suggestive of cholecystitis.
    _________________________________________________

    XraySkullArrowGIF 1 - The Medical student Review
    skull x ray
    can detect fractires, ltic and blastic lesions frm bony mets or paget dz such as in above.
    they are incapable however of detecting a bleed.
    look for pt who has experienced head trauma. but it is almost always the wrong answer..
    >>>>>>>>>>>>>>>>>>>>>>>>>>>>>>>>>
    Last edited by جوري; 10-05-2008 at 03:59 AM.
    The Medical student Review

    Text without context is pretext
    If your opponent is of choleric temperament, seek to irritate him 44845203 1 - The Medical student Review


  19. Report bad ads?
  20. #35
    جوري's Avatar Full Member
    brightness_1
    Soldier Through It!
    star_rate star_rate star_rate star_rate star_rate star_rate star_rate star_rate star_rate star_rate star_rate
    Join Date
    Jul 2006
    Location
    من ارض الكنانة
    Gender
    Female
    Religion
    Islam
    Posts
    27,759
    Threads
    1260
    Rep Power
    258
    Rep Ratio
    89
    Likes Ratio
    23

    Re: The Medical student Review

    10-5-5

    heme

    pt comes to the office because of fatigue and slowly progressive dyspnea on exertion. the HCT is low at 25% and the MCV is markedly elevated at 130fl. the peripheral smears shows hypersegmented neutrophils with an average of 5 lobes. The LDH and indirect bilirubin are also elevated. the retic count is low.
    DX,
    1- elderly pt with glossitis and peripheral neuropathy. both the methylmalonic acid and homocysteine levels are elevated.
    2-malnourished alcoholic with an elevated homocysteine level

    Vitamin B12 deficiency and folic acid deficiency are identical in their hematologic abnormalities. The most common abnormality of B12 deficiency is peripheral neuropathy.
    2- Folic acid deficiency does not give neurological abnormalities. in addition folic acid deficiency elevates only the level of homocystine, whereas B12 elevates both homocysteine and the level of methylmalonic acid.

    _______________________________________________

    pt with sickle cell dz is admitted because of fatigue developing over several days. The HCT has dropped precipitously. the MCV is normal and the retic count is low. WBC count and plt are normal
    dx, accurate, initial therapy

    Parvovirus B19 is the most likely cause of pure Red-Cell aplasia in a person with a hemoglobinopathy. The retic count should be elevated in a person with anemia. This is particularly true in the case of sickle cell dz in which the retic count is usually 10-20%.
    Parvo-virus invades the bone marrow and freezes the growth of precursor cells in the marrow. the retic count is abnormally low. Because a pt with SCD has such a high percentage of reticulocytes. the HCT can drop very precipitously when infection with parvovirus occurs.

    the most accurate test for B19 is PCR for DNA. If this is not one of the choices, then test is IgM against the virus. Although bone marrow biopsy with an increased number of giant pronormoblasts does develop. this is obviously more invasive and not as specific as the PCR for parvovirus DNA.

    RX is with IV immunoglobulins.

    _______________________________________________
    a pt comes to the er with sudden onset fatigue, SOB and HCT 20%. MCV is slightly elevated, the retic count, LDH and indirect bili are all elevated and the haptoglobin is low

    1-HX of SLE. CLL, lymphoma or med such as PCN
    2-Recurrent episodes with a large spleen. Often with fam hx. An elevated MCHC
    3-Sudden onset of hemolysis in a male pt with an acute infection. occasionally happens after sulfa drug use.

    1- All forms of hemolysis, lead to elevated LDH, indirect bili and retic. Autoimmune warm antibodies are found in association with SLE, lymphoma, and CLL. In addition, medications such as PCN, sulfa, and quinidine can provoke autoimmune hemolysis lead to elevated levels of LDH, indirect bili, and retic. Autoimmune warm antibodies are found in association with SLE, lymphoma, and CLL. In addition, medications such as PCN, sulfa medications and can provoke autoimmune hemolysis. The Most accurate test i a coombs test. the smear will be normal because the hemolysis is occurring in the spleen.
    2-Hereditary spherocytosis presents as recurrent episodes of hemolysis. The MCHC is elevated because red cell membrane is to tight to contain the amount of hemoglobin present. The most accurate test is osmotic fragility.
    3-Glucose-6-phosphate dehydrogrenase (G6PD) deficiency presents most often in males because it is X linked. Although primaquine, dapsone and fava beans have been classically associated with this disorder, the most common cause of acute hemolysis is an infection. the best initial tests are for Heinz bodies and bite cells.
    __________________
    A man comes to the office wit dark urine in the morning, his urinalysis shows hemoglobin but no red cells are visible, there are no white cells or protein. the CBC shows anemia and mild thrombocytopenia, the LDH, indirect bilirubin and retic count are elvated. He has a HX of a large vessle thrombosis. The Leukocyte alakaline phosphatase level is low.

    DX, dignostic and common cause of death

    PBH presents with episodes of dark urine in the morning. The hemolysis occurs over night with hemoglobin visible with the first morning urine. Pancytopenia often presents as a sign of hemolysis such as elevated bili, and retic count are present. the retic count maybe low.. A low Leukocyte alkaline phosphatase maybe present.
    the most accurate test is a CD55/59 antigen test that is low. CD55/59 is the marker for decay accelerating factor. DAF removes complement from the cells before they are destroyed. older less accurate tests are sugar/water and Ham's test which look for activation of complement.

    The most common cause of death is large vessel thrombosis. less common complications are acute leukemia, aplastic anemia, and myslodysplasia. This is because PNH is a stem cell disorder.

    __________________________________________________ _____

    An african american man comes to the ER wth pain in his back, chest and thighs, he has a hx of sickle cell disease. He is febrile 102f. cxray and urinalysis normal. o2, fluids, and analgesics are started his HCT is 28%

    most urgent ste
    best initial test to confirm parvo B19
    most accurate diagnostic
    initial to confirm SCD

    the most urgent step in SCD when a fever is present is to start antibiotics such as ceftriaxone, levofloxacin or gatifloxacin. Don't wait for results of cultures. pts with SCD can die rapidly of overwhelming sepsis because they are functionally asplenic.
    pavovirus B19 results in aplastic crisis particularly in those with an HX of hemoglobinopathies. The best initial test is a reticulocyte count. pts with SCD usually have a high retic count. PARVOVIRUS gives a low retic count.
    the most accurate test for parvo is a PCR for the DNA virus.
    the best initial test for sickle cell dz is a peripheral smar. the most accurate test is a hemoglobin elecrophoreis. pts with SCD often present with acute chest syndrome...

    >>>>>>>>>>>>>>>>>>>>>
    The Medical student Review

    Text without context is pretext
    If your opponent is of choleric temperament, seek to irritate him 44845203 1 - The Medical student Review


  21. #36
    جوري's Avatar Full Member
    brightness_1
    Soldier Through It!
    star_rate star_rate star_rate star_rate star_rate star_rate star_rate star_rate star_rate star_rate star_rate
    Join Date
    Jul 2006
    Location
    من ارض الكنانة
    Gender
    Female
    Religion
    Islam
    Posts
    27,759
    Threads
    1260
    Rep Power
    258
    Rep Ratio
    89
    Likes Ratio
    23

    Re: The Medical student Review

    yesterday was a disaster I set out to do 10/5/5 did only five.. the ac adapter of my new computer is dead.. finito so here we are on desktop.. let's make up what was lost..


    Heme in most likely DX

    1-An AA is taking a course in skydiving, he is on his first time up at high altitude, about to jump when he develops severe chest, back and thigh pain. When the plane returns to the ground, he feels well, his CBC including, peripheral smear, isnot normal. His only medical hx is of dark urine
    dx, and diagnostic?

    Sickle cell trait or hetrozygous (AS) sickle cell disease is present in 8% of African Americans. Acute painful crisis in SCT is extremely rare and occurs and occurs only under conditions of the most severe hypoxia or high altitude, such as would occur during parachute jump. The only significant manifestations of AS disease are renal concentrating defect (isosthenuria) and occasional episodes of gross hematuria. There is no specific therapy.
    Most accurate test for ickle cell trait is a hemoglobin electrophoresis. there is no specific therapy!

    __________________________________________________

    2-A man from Miami recently moved to Chicago for his residency. He has an episode of pneumonia that is mild with a dry cough and bilateral interstitial infiltrates that resolve with azithromycin, while shoveling snow he suddenly develops pain and discoloration of his fingers, nose an ears. His hct is 28%, and the bilirubin, LDH and retic count are elevated.
    dx, accurate test, therapy

    Cold agglutinin disease or igM-induced antibodies is the most likely diagnosis when there is hemolysis in association with pain and discoloration of acral portions of the body such as the fingers, nose and ears on exposure to cold. in addition, although most cases of cold agglutinin disease are asymptomatic, look for a recent history of mycoplasma pneumonia such as suggested in this case. Epstein barr virus is also another clue.
    the most accurate test is a direct coombs that is positive for complement only. all the usual findings of hemolysis are present, such as an elevated LDH , indirect bili, and retic count but they are nonspecific for cold agglutinin disease

    no specific therapy is usually necessary, steroids aren't helpful. in severe cases, alkylating agents such as cyclophosphamide can be used. cyclosporine are also helpful.

    ________________________________________________

    3-A man with diarrhea comes because of weakness and anemia, in addition, he has an elevated retic count, LDH, and indirect bili level. The haptoglobin is absent. the plt count is 38,000 but he isn't bleeding. the creatinine is 2.4

    dx,cause of diarrhea, best test, most accurate test

    Hemolytic uremic syndrome (HUS) is the triad of hemolytic anemia, renal insufficiency and thrombocytopenia. If neurologic abnormalities and fever are also present, this is thrombotic thrombocytopenic purpura TTP

    HUS is often associated with E.coli 0157:H7.
    the best initial test is a peripheral smear showing fragmented red cells such as schistocytes or helmet cells pls see previous page for slide. This is also referred to as microangipathic hemolytic anemia.

    there is no specific test for either HUS or TTP. they are diagnosed bases on either the triad or pentad of laboratory abnromalities. Most of the time they will resolve spontaneously. Do not give plts or antibiotics.

    ______________________________________________

    4- Man comes to the office with sx of dizziness, headache, fatigue, and blurred vision, he is very itchy after a warm shower, he gets nosebleeds, he has splenomegaly. His hematocrit is 58%. The MCV is low at 68fl the white cell count and plt are normal.
    dx,best initialt diagnostic, most accurate, and common cause of death..

    polycythemia vera is a neolasm of the bone marrow with markedly elevated hct. in the absence of hypoxia or an elevated level of erythropoietin. Polycythemia vera presents with signs of hyperviscosity such as headache, blurry vision, fatigue and epistaxis. pruritis after a warm show is common because of histamine release from basophils. the cells in polycythemia vera are small.

    the best initial test is an arterial blood gas to exclude hypoxia as a cause of secondary polycythemia. if the hematocrit is markedly elevated above 60% and the white count and plt count are elevated. no additional tests besides a bone marrow biopsy are necessary because nothing else besides polycythemia vera will give an elevation of the three cell lines.

    the most accurate is a nuclear red-cell mass test
    the most common cause of death is large vessel thrombosis from hyperviscosity of elevated red cell mass.

    ____________________________
    5- 52 year old man comes to the office with painful burning in his hands. the only lab abnromality is plt count of 1,500,000
    dx, accurate diagnostics, common cause of death, best therapy


    RM3 - The Medical student Review
    is a platelet cancer. this is a myeloproliferative disorder of the bone marrow in which the platelets are elevated to levels above 1 million. The white cell count can also be up.
    ET ca present with erythomelalgia, which is painful, red burning of the hands; however, it may present with a high platelet count only.
    there is no specific test, the bone marrow shows, nothing except increased number of megakaryocytes. Red cells are normal, there is a high frequency of mutation to JAK2.
    essential thrombocytosis can result in death from either bleeding or thrombosis. thrombosis is more common.
    the best initial therapy is hydroxyurea.

    __________________________________________________

    6-an elderly man is being evaluated for progressive fatigue. CBC shows pancytopenia. MCV is normal

    dx
    1-splenomegaly, nucleated red cells, rear drop cells, and leukoerythoblastosis.
    2-splenomegaly, a noaspirable dry tap, and a positive tartarate resistant acid phosphatase
    3-pancytopenia alone with a vacant bone marrow

    1-myelofiboris is diagnosed by finding the combination of nucleated red cells, teardtop-shaped cells, and an immature white cel that forms on smear such as promyelocytes or myeloblasts. All together, this is caled 'leukoerythoblastic' presentation. the liver and spleen are big because progressive marrow fibrosis leads to extramedullary erythropoiesis.

    2-hairy cell leukemia wwwislamicboardcom - The Medical student Review

    presents in middle aged ts with pancytopenia, massive splenomegaly, and a dry tap. the most accurate test is the TRAP or tartarate-resistant acid phosphatase.

    3-Aplastic anemia is simply pancytopena of unclear etiology. the marrow is empty and can be replaced by fat, there is no fibrosis in the marrw. splenomegaly is not present because extramedullary hematopoiesis is not occurring. the marrow has just simply died. Remember parvovirus B19 in pts with previous baseline anemia such as sickle cell, or thalassemia can cause transient aplastic anemia.

    __________________________________________________ _
    7- a 60 year old man comes in with fatigue, low grade fever and abdominal fullness, massive splenomegaly is found. His white cell count is markedly elevated at 175,000, they are normal and mature appearing on smear. the leukocyte alkaline phosphatase (LAP) score is low
    dx, accurate diagnostic and common cause of death

    Chronic myelogenous leukemia (CML) presents with fatigue, upper abdominal pain

    page3 - The Medical student Review

    bonemarrow1 1 - The Medical student Review

    presents with fatigue and left upper quadrant pain from really big spleen. the white cell count is markedly elevated but they look normal and the LAP score is low. A low LAP scores means that the cells maybe up in number but they are low in function.

    the most accurate test for CML is the philadelphia chromosome. this can be called bcr/abl mutation.
    cml 3 - The Medical student Review\

    without rx with imatinib, 20% of CML pts will transofmr into acute myelogenous leukemia each year.

    __________________________________________

    8- a 72 year old man comes in for progressive fatigue, he has splenomegaly on examination, his HCT is 30% with an MCV of 107fl. There are oval shaped cells. the retic count is reduced. the white cells show bilobed nuclei. There is a mild reduction in plt count. B12 and folate levels are normal.

    dx, accurate diagnostic

    myelodysplastic syndromes are a collection of pre-leukemic syndromes with macrocytic anemia, they are seen exclusively in elderly pts. they often have a bilobed neutrophils known as Pelger-Huet cells
    hem i 5PelgerHuetCell - The Medical student Review
    the platelet countand retic count are often reduced. although a small number of pts progress to acute myelogenous leukemia most pts will die of bleeding or infection before that occurs.

    the most accurate test is a bone marrow biopsy. the marrow is hypercellular despite the peripheral low counts. the prussian blue shows ringed sideroblasts .
    sideroblast 1 - The Medical student Review

    ___________________________________________
    9- 34 year old man comes in with severe bleeding from his skin, nose and rectum, he has a fever, CBC shows pancytopena, there are blasts visible on the peripheral smear. The PT and PTT are elevated. Some of the neutrophils have an eosinophilic inclusion body visible.

    dx, diagnostic

    acute promyelocytic leukemia, or M3 leukemia, presents with the same pancytopenia as any other acute leukemia with blasts present. in addition, promyelocytic is always the most commonly asked question, because it is acute and has a very distinct presentation. The association with disseminated intravascular coagulation (DIC) is characteristic or promyelocytic leukemia. , this is because promyelocytic granules activate the coagulation pathway
    0062 1 - The Medical student Review
    the esopsinophilic body is an Auer Rod, which is characteristic of promyelocytic leukemia ( also AML)

    the most accurate test for acute leukemia is a bone marrow biopsy. this is the most accurte way to assess the number of blasts, in addition, the most important test to determine prognosis are cytogenetic studies. these are best obtaines on actively replicating cells found in the marrow.

    ________________________________________________
    10- a 60 year old man is found to have an elevated total protein on routine blood testing in the office. Electrophoresis reveals a monoclonal IgG spike. Calcium, CBC, urinalysis, and skeletal bone survey are all normal
    dxm accurate diagnostic, initial therapy

    this is monoclonal gammopathy of unknown significance (MGUS) is most found on routine testing of blood protein in an elderly pt. The pt is asymptomatic. all other tests are normal. There will be no Bence jones protein, no bone lesions and a normal uric acid level.
    the most accurate test is a bone marrow biopsy, MGUS has <10% plasma cells on bone marrow biopsy.
    there is no therapy for MGUS. only 1% of pts per year will progress to myeloma, and no therapy is known to prevent this.

    __________________________________________

    pharm.
    a young woman with multiple bowel movements per day and abdominal pain, some BMs have blood, she was dxed with crohn's disease
    Mesalamine derivatives are the best initial therapy for both crohn's and UC. Mesalamine is the most effective way of delivering 5-aminosalicylic acid (5-ASA) several formulations exist.
    Asacol delivers the 5-ASA to the distal bowel
    Pentasa delivers therapy to the entire bowel.
    sulfasalazine gives 5-ASA but has many more adverse effects such as rash, hemolysis and renal tox, because of the sulfa component.
    acue episodes of worsening inflammatory BD that are not controlled with 5-ASA derivatives are treated with oral steroid budesonide. Budesonide has enormous first pss effect on the liver, so it has limited toxicity. Azathioprine can be used to keep the disease nder control without the use of steroids.

    _____________________________________________

    44 year old woman comes for a follow up, she has has acid reflux for 5 yrs. Endoscopy reveals a 3 cm of columnar epithelium of the distal esophagus.

    barret's esophagus is columnar epithelium extending up out of the stomach into the distal esophagus. Barrett's esophagus should be treated with a PPI. such as omeprazole, pataoprazole, lansoprazole, esmeprazole, or rabeprazole.

    PPIs work by inhibiting the release of acid from the parietal cells of the stomach. they inhibit the K+/H+ ATpase. PPIs have no major adverse effects.

    surveillance endoscopy for Barretts esophagus should be performed every 2-3 yrs. if low grade dysplasia is present, repeat endoscopy every 6 months.

    ____________________________________________

    a man develops multiple ulcers in his duodenum. they recur after rx for H pylori. his gastrin level is elevated. he undergoes endoscopic US and an octreotide scan to determine therapy

    an endoscopic US and octreotide scans are performed to determine the presence of mets. localized ZES or gastrinoma is treated by surgical resection
    Metastatic ZES is treated with lifelong PPI.

    _____________________________________________
    a 55 yo woman comes in with pruritis, her alk phosp is elevated, her AST and ALT are minimally elevated. anti-mitochondrial antibodies are present in increased amount.

    this is primary billiary cirrhosis, treated with ursodeoxycholic acid. Cholestyramine maydecrease itching but should not be given at the same time as ursdeoxycholic acid, because it may decrease absorption.

    the mechamism or ursdeoxycholicacid is poorly understood, it decreases plasma and endogenous bile acid conc. this reduces hepatotoxicity, because endogenous bile acids are more toxic to the liver. ursodeoxycholic acid decreases esiosinophil activation and may decrease the immune destruction of hepatocytes.
    urodeoxycholic acid generally devoid of adverse effects.

    __________________________________________________ ____

    43 year old man comes with patiguem joint pain, darkened skin, ED, serum iron and ferritin levels are markedly elevated. the MRI shows abnromal liver.

    2- pt with SCD comes in for rx, she has been transfused for severe sickle cell crisis 5 tomes a year for 10 yrs

    case one is heriditary hemochromatosis from overabsorption of iron in the duodenum. the pt should undergo periodic phlebotomy to dec iron overload.

    case two has developed hemochromatosis from repeated transfusion. can't use phlebatomy on someone who is anemic, thus chelation therapy is used. Deferasirox is an oral iron chelator, Deferoxamine has to be given by injection and is harder to use
    chelating agents bind to iron and allow it to be excreted..

    ________________
    diagnostics

    pelvic us, vaginal US

    Pelvic US used for
    pelvic or lower abdominal pain with amenorrhea or vaginal bleeding
    suspected pregnancy
    suspected ectopic pregnancy

    transvaginal US
    superior to pelvic used in
    placenta previa (painless vaginal bleeding in the third trimester)
    fetal monitoring in the first trimester
    obese women in examining pelvic anatomy.

    ____________________________
    CalciumOxalateCrystalsbipyramidalRhubarb 1 - The Medical student Review

    the crystals are enveloped shaped oxalate crystals
    they precipitate in renal tubules when toxic dose of ethylene glycol, a component of antifreeze is ingested.
    most specif test is to determine blood level. urine can be examined under fluorescent wood's light, however, urinary fluroescene lasts only a few hours after ingestion. bcause the oxalic acid precipitates with calcium the blood level of calcium is always low.
    answer etheylene glycol overdose and oxalate crystals when you see an elevated anion gap and metabolic acidosis . the question most often describes a depressed pt with possible attempted suicide, Hypocalcemia can lead to QT prolongation and cardiac arrhythmias. Oxalate crystal deposit in the renal tubules can cause acute tubular necrosis.
    _____________________________

    Anti-basement membrane antibodies (Anti-BM)
    are indicative of goodpasture syndrome

    goodpasture - The Medical student Review

    pt with hematuria, and HTN, accompanied by a cough and SOB, hemoptysis. Renal insufficiency and red cell casts in the urine are characteristic..
    the most accurate test for goodpasture is a lung or kidney biopsy as in above.
    ___________________________

    total complement level (ch50)
    complement deficiency leads to recurrent bacterial infections. CH 50 or total complement assay, is performed on young pts with recurrent unexplained infections such as pneumonia, sinusitis, meningitis with encapsulated organisms such as Neisseriae, s. pneumonia, and H flu. the pt will have normal lymphocyte and immunoglobulin levels. an abrnomal CH 50 assay indicated inherited or acquired complement deficiency.

    ______________________________________
    urine microalbumin

    this is a sensitive screening test for diabetic nephropathy

    look for a case with diabetes for >5 yrs who has normal renal function and a negative urinalysis for protein. if the dipstick is already positive for albumin, there is no reason to look gor microalbuminuria. screening is done on yearly basis .

    if the test is positive use ace inhibitors, or ARBs to prevent rogression of disease.
    a negative test for microalbumin is <30mg/day, levels between 30-300md/day are called microalbuminuria. levels above 300 would register positive on the normal urinalysis.
    renal biopsy is the only test of the effect of diabetes on the kidney that is more sensitive than microalbuminuria.

    ___________________________________
    Last edited by جوري; 10-07-2008 at 04:19 AM.
    The Medical student Review

    Text without context is pretext
    If your opponent is of choleric temperament, seek to irritate him 44845203 1 - The Medical student Review


  22. #37
    جوري's Avatar Full Member
    brightness_1
    Soldier Through It!
    star_rate star_rate star_rate star_rate star_rate star_rate star_rate star_rate star_rate star_rate star_rate
    Join Date
    Jul 2006
    Location
    من ارض الكنانة
    Gender
    Female
    Religion
    Islam
    Posts
    27,759
    Threads
    1260
    Rep Power
    258
    Rep Ratio
    89
    Likes Ratio
    23

    Re: The Medical student Review

    a 70 year old man comes to the hospital with blurry vision, SOB, confusion, vertigo and nausea, He is anemic and the white cell count is normal. The serum viscosity level is increased to 1.5 times that of water. He has engorged sausage shaped blood vessels in his eyes
    dx, diagnostic

    Waldenström's macroglobulinemia ( lymphoplasmacytic lymphoma)
    microexambm42 - The Medical student Review
    is caused by hyperviscosity from the overproduction of IgM from lymphocutes and plasma cells. IgM is larger than IgG and therefore presents with a hyperviscosity syndrome that obstructs blood vessels in the brain, lungs, and eyes and results in SOB, vertigo also occurs. GI bleeding may occur from engorged vessels. This is the same short of presentation as leukostasis reaction in acute leukemia, however, the white-cell count in Waldenström's macroglobulinemia is generally normal.

    the most accurate test is a serum protein electeophoresis with an elevated IgM spoke and a bone marrow biopsy showing increased plasma cells. Bone x-rays are normal.

    _____________________________________________

    a pt. is admitted for a PE. two days after starting IV heparin the plt count starts to dec.
    dx, diagnostic

    heparin indusced thrombocytopenia (HIT) occurs several days after the start of heparin. The most common presentation is an asymptomatic decrease in the plt count. Occasional episodes of thrombosis may occur. Venous thromboses are generally three times more common than arterial thrombosis. In general, a 50% decrease in the number of platelets after starting heparin is considered a threshold criteria.
    The most accurate test for HIT is for antibodies to platelet factor 4. these are heparin induced anti platelet antibodies. Serotonin release is very sensitive. RX is to sop all heparin products immediately.

    _______________________________________________

    Pt is admitted to psych for an acute episode of hallucination, psychosis and hysteria, she also has abdominal pain and dark urine. She recently started phenobarbital for seizures. The attack accompanied the onset of menses. Despite the severity of her abdominal paon, the exam is benign.

    dx, initial diagnostic, initial therapy

    acute intermittent porphyria (AIP) presents with severe abdominal pain, neuropsychiatric disturbance, and dark urine. Episodes often happen around menses and or after the start of meds such as phenobarbital.
    AIP is confirmed with urinary levels of aminolevulinic acid and porphobilinogen.

    AIP is treated acutely with dextrose and IV heme infusion.

    ____________________________________

    a woman comes in with increased bleeding after a dental extraction. she has noticed increased bleeding such as epistaxis and petechiae for many yrs. The plt count is normal. the aPTT is modestly elevated.
    dx, diagnostic, most accurate

    Von Willebrand's disease presents with an increased mucosal type of bleeding, particularly after minor trauma, surgery, or ASA use.
    the APTT can be elevated because factor VIII antigen (von willebrand's factor, VWF) and factor VIII coagulant ( hemophilia A factor) travel bound to each other. This can't be hemophilia because the type of bleeding is hemophilia would be deep bleeding into a joint or into muscle, such as a hematoma. in addition hemophilia does not express itself in women.
    the best initial test for plt function is bleeding time. Do not do a bleeding time is the plt count is low. if the plt count is low, the bleeding time will be abnormal.

    the most accurate test for von willebrand's dis is a combination of VWF and ritocetin testing. Ristocetin testing determines the function of the VWF if the level is normal.

    ________________________________

    a pt comes in with bleeding into his joints after minor muscle trauma. the plt count is normal.

    1-male child with an elevated aPTT and a normal PT
    2-Pt who has recently has an IV antibiotics. There is elevation of both PT and aPTT.
    3-an alcoholic pt with low albumin, who also has varices. both PT and aPTT are elevated.

    1- Hemophilia is the most likely dx with hemarthrosis n a male child after a minor trauma. only the aPTT will be elevated. The best initial test is a mixing study and the most accurate is a specific level of factor VIII or IX. The mixing study s the first test to perform to determine the presence of a clotting factor deficiency. if the aPTT is elevated from a clotting factor deficiency, the lab value will return normal when mixed 50:50 with normal plasma. if there is a clotting factor inhibitor, it will not correct.
    2-Vit K deficiency is suggested by the recent antibiotics which deplete the levels of vit K in the body. Both PT and aPTT will be elevated. The diagnosis confirmed by looking for an improvement after administering supplementary vit K.
    3-Liver disease presents in the same manner clinically as vit K deficiency, but there will be no improvement after adding supplementary vit K.
    __________________________________

    a pt comes in with a elevated aPTT found on routine screening prior to a minor procedure. The PT is normal
    1- There is no bleeding at any time, the pt completely asymptomatic.
    2- the pt has minor bleeding occasionally in the past, but only with trauma or surgical procedures.
    3-there has been clotting such as a DVT, in the past. The VDRL is positive.

    1- Factor XII deficiency produces an elevation in the aPTT with no evidence of bleeding even under conditions of additional trauma.
    2-Factor XI deficiency results in a prolonged aPTT and gives abnromal bleeding under conditions of trauma or surgery. such as a dental extraction. Factor 11 deficiency is common in Ashkenazi Jews.

    3-Lupus anticoagulant is a type of antiphospholipid antibody that results in increased clotting but gives a prolonged aPTT as a lab artifact . It is associated with a false positive VDRL. on mixing studies, the aPTT will not correct on a 50:50 mix normal plasma because it is a circulating antibody. the antibod will be presents in the mix. Deficiencies correct to normal when mixed, antibodies will not.
    ________________________________________

    a pt is admitted with a pulmonary embolus, he is not obese or elderly, there is no malignancy or increased risk of clotting that can be identified.
    1-Most common cause of thrombophilia
    2-there is skin necrosis with the use of warfarin
    3-the aPTT does not rise after the use of heparin

    1- Factor V leiden mutation is the most common cause of thrombophilia. This is a genetic defect that results in resistance of factor V to inactivation by protein C.

    2- Protein C is associated with skin necrosis with the use of warfarin. protein C is a natural vitamin K-dependent anticoagulant with a very short half life. When starting warfarin, there is a transient hypercoagulable state that is produced for a short time before the other clotting factors are inhibited.

    3-antithrombin III deficiency is a cause of thrombophilia that results in resistance to heparin. Heparin works by potentiation of the effect of antithrombin. If there is a low level of antithrombin then heparin will no work. There will be no rise on aPTT afte a bolus of heparin.

    _____________________________________________
    pls diagnose each of the following transfusion reactions

    1-Mild febrile reaction with the first unit of blood. with the second unit there is SOB and Pulmonary infiltrates that resolve in 24hrs
    2-immediate anaphylaxis after transfusion
    3-mild urtricarial reaction after transfusion, no evidence of hemolysis
    4- A single degree centigrade with no evidence of hemolysis.

    1-Leukoagglutnination reaction occur from donor antibodies attacking and agglutinating recipient WBC, resulting in SOB, this is also known as transfusion associated ling injury (TRALI) no rx necessary.

    2-IgA deficiency leads to anaphylaxis and occurs from IgA in the donor blood. This occurs in IgA deficient recipient. Use blood from IgA deficient donors in the future.
    3-Urticarial rxn occur as an allergic rxn to donor plasma proteins. urticarial reactions can be prevented by transfusing washed red cells.
    4-febrile non-hemolytic reactions occur from a reaction against donor white cells. prevent this by filtering blood.

    ________________________________________________

    a young pt comes in with multiple sinopulmonary infections. He has had sinutitis, bronchitis, pneumonia and otitis media

    1- the pt is an adult with normal lymph nodes. B-Cell numbers are normal. Immunoglobulin produced is markedly low.
    2- A male child has infections in the first year of life, lymph nodes and tonsils are hypoplastic, Immunoglobulin levels and B cells are absent.
    3-T cells are absent. there are cardiac defects, facial abnormalities, and hypocalacemia with low PTH level IgG levels are normal

    1-Common variable immunodeficiency (CVID) presents in adults with normal numbers of B cells but markedly low immunoglobulin levels.
    RX is with replacement of immunoglobulins.
    2-X-linked agammaglobulinemia (Bruton's) presents in make children at an early age. Not only is no immunoglobulin produces, but the B cells and normal lymphoid structures are missing. RX is with immunoglobulin replacement.
    3-Di George's syndrome is an isolated T-Cell deficiency from thymic hypoplasia. DiGeorge's syndrome is associated with cardiac and facial defects. hypocalcemia results in the inability to develop parathyroid glands. Bone marrow transplant is used in severe cases.
    _____________________________________________
    a young pt comes in with repeated episodes of otits mesia and pneumonia. in addition there is eczema and dermatitis.

    1- Allergic disorders, asthma and urticaria also occur. There was a severe persistent diarrheal illness from Giardia lamblia. A blood transfusion resulted in anaphylaxis.
    2-a male child presenting at a very early age with infections also has a bleeding disorder. the platelet count is low and platelets are small in size.

    IgA deficiency most commonly comes to attention because of frequent sinopulmonary infections. There are multiple allergic disorders with IgA deficiency such as asthma, urticaria, rhinitis and atopic eczema. Chronic infections can result in anaphylaxis. i the cells are not washed because of an allergic reaction to IgA in the donor blood. There is no specific therapy. Transfusion should only be from IgA deficient donors or with washed red cells.

    2-Wiskott-Aldrich syndrome is the combination of increases susceptibility to infection combined with eczema and thrombocytopenia. Atopic dermatitis occurs with increased frequency of otitis media, pneumonia, and thrombocytopenia. bleeding is common.

    _______________________________________
    \pharm

    25 year old woman comes to see you for the fifth time in two months because of abdominal pain, she has periods od diarrhea alternating with constipation, the sx are less severe at night. Her abdominal CT scan and colonoscopy are normal.

    IBS is initially treated with dietary modifications such as increasing fiber content. the best sources of fiber are bram, psyllium, husks, and methcellulose.
    Antispasmodic medications such as dicyclomine, or hyoscamine may decrease the pain, they have modest anticholinergic effect. diarrhea can be treated with dihenoxylate or loperamide which inhibit gi motility.
    if fiber and antispasmodic agents aren't effective, tricyclic antidepressants may work, tricyclics are anticholinergic and have a beneficial effect of neuropathic pain.

    ______________________________________________

    Sildenafil, tadalafil and vardenafil
    these are the best initial therapies for erectile dysfunction. sildenafil has also been approved for rx of pulmonary htn.
    these meds are phosphodiesterase inhibitors. they decrease vasculat tone. and increase flow into the penis by increasing local concentrations of NO
    phosphodiesterase inhibitors can cause hypotension, headache and facial flushing. they should not be used in combo with nitrates for pts who have CAD. grapefruit juice can increase the levels of these meds because of its effect on inhibiting cytochrome p450.

    __________________________________________________ _

    a 72 year old man comes to the office because of a delay in his ability to urinate. He has a decreased urinary stream, increased urinary fequency, there is no burning pain when he urinates. urinalysis is normal.

    the best therapy for benign prostatic hypertrophy is a combination of a 5 alpha reductase inhibitors, such as finersteride, and peripheral alpha blockers, such as doxazosin, prazosin or tamsulosin.
    finersteride blocked production of dihydrotestersterone, which stimulates growth of the prostate, opening the bladder neck, prostate capsule, and prostatic urethra, prazosin, terazosin, doxazosin and tamsulosin increase the volume of urinary stream.
    peripheral alpha blockers can cause dizziness and orthostatic hypotension, Tamsulosin has the same therapeutic benefit with less incidence of hypotension.
    if medical therapy fails, the treatment of BPH is a transurethral resection of the prostate TURP.

    _________________________________________

    Finasteride and dutasteride
    are te best therapies for benign prostatic hypertrophy. they decrease the volume of the prostate and increase the volume of urinary flow. Finasteride is also approved to treat male pattern hair loss.
    hese meds inhibit 5 alpha reductase, which converts testosterone to dihydro-testosterone. dihydro-testosterone is responsible forthe growth of the prostate as well male pattern baldness.
    Finasteride and dutasteride have a very small effect of decreasing libido, decreasing the volume of ejaculate, and causing a mild increase in the incidence of erectile dysfunction.
    ________________________________
    ezetimibe

    is used to rx hyperlipidemia. it is added to HMG-CoA reductase inhibitor (a statin) if the low density lipoprotein (LDL) is not sufficiently controlled. The major advantage of ezetimibe it is limited side effects.
    Ezetimibe inhibits absorption of cholesterol at the brush border it blocks the sterol transporter.
    Ezetimibe causes diarrhea and abdominal pain because of its ability to block fat absorption.

    ____________________________

    diagnostic

    water deprivation test

    is used to distinguish between the causes of diabetes inspidius (DI) to objectively assess pts with hypernatremia and high urine output.
    you restrict water intake for 12-24hrs with observation for the volume of urine, urine osmolality, and change in body weight to evaluate whether the kidneys are conserving or excreting water properly, if the urine volume stays elevated after the serum osmolality goes up, then it is DI. If there is a decrease in response in response to giving vasopressin or ADH, then it is central DI, if no response then it is nephrogenic

    if this is a bit confusing to you, and you like anecdotal learning, then I have covered the tread entirely earlier here
    http://www.islamicboard.com/health-s...dus-siadh.html

    ________________________________________

    artsm448502fig2 1 - The Medical student Review

    diascopy is the best initial test for erythematous lesions. It is a fancy way to see of the lesion blanches (turns white), a lear plastic slide is pressed over the skin, used to determine if an erythmatous skin lesion is due to vasodilation or extravasation of RBCs via damaged blood vessels.

    if the skin blanches with diascopy, the erythema is causes by vasodilation, simple vasodilation is caused by viral diseases, drug reactions and insect bites. if the lesion persists under diascopy, the redness is caused by extravsation of erythrocyte via damaged blood vessles, as seen with vascuilitic and purpuric lesions.

    __________________________________________________ ____

    melanoma2 - The Medical student Review
    excisional biopsy is a skin technique where the full thickness of the lesion is removed preferably intact.
    this is best for such and malignant melnoma see above.
    can't be staged properly otherwise is not fully removed.

    _____________________________________________

    direct immunofluorescence (DIF) of skin biopsy samples is used to diagnose skin disorders of immunologic origin. The DIF test looks for the presence and staining pattern of immunoglobulins 9IgG, IgM, IgA), C3 and fibrinogen in the skin. for examle it is useful for differentiating between pemphigus vulgaris and bullous pemphigoig
    pemphingus - The Medical student Review
    pemphigus vulgaris immunofluorescence ( IgG and C3 between epidermal cells)
    artdn470164fig1b 1 - The Medical student Review

    bullous pemphigoid immunofluorescence (IgG and C3 at dermodermoepidermal junction)

    ________________________________

    fungal culture
    is the most accurate diagnostic test superficial fungal infections of the body. examples, tinea, onychomycosis and fungal vaginits.
    not the best initial test though, fungal infections are primarily diagnoses by visual appearance and confirmed by KOH prep.
    onychomycosis (nail bes) and tinea capitus (scalp) need to be treated with oral antifungals. (terbinafine or itracaonazole). all of the other fungal infections of the skin may be treated wit topical meds such as ketoconazole, clotrimazole, nystatin or ciclopirox.

    >>>>>>>>>>>>>>>>>>>>>>>>>>>>>>>>
    Last edited by جوري; 10-08-2008 at 03:41 AM.
    The Medical student Review

    Text without context is pretext
    If your opponent is of choleric temperament, seek to irritate him 44845203 1 - The Medical student Review


  23. #38
    جوري's Avatar Full Member
    brightness_1
    Soldier Through It!
    star_rate star_rate star_rate star_rate star_rate star_rate star_rate star_rate star_rate star_rate star_rate
    Join Date
    Jul 2006
    Location
    من ارض الكنانة
    Gender
    Female
    Religion
    Islam
    Posts
    27,759
    Threads
    1260
    Rep Power
    258
    Rep Ratio
    89
    Likes Ratio
    23

    Re: The Medical student Review

    we start with infectious disease in most likely dx

    1- pt comes in with fever, headache, and vomiting, he experiences a seizure..

    1- confusion is the main compliant
    2-stiff neck (nuchal rigidity) and photophobia are present.
    3-He has focal neurological deficits and projectile vomiting.

    1- encephalitis is characterized predominately by confusion and fever for a few days. Although there is a headache, nausea, vomiting and seizures. these findings aren't specific to encephalitis. Encephalitis is best diagnosed with a head CT followed by a lumbar puncture. The most accurate diagnostic test for herpes encephalitis is a PCR of the CSF, not a brain biopsy.

    2-meningitis presents with stiff neck and photophobia

    3- brain abscess presents with focal neurological findings in addition to confusion, and focal findings--

    ______________________________

    pt comes to the ER dept with dever, headache, neck stiffness and photophobia
    1- six hours of sx with 3,500 white cells that are predominantly neutrophils.
    2-neutrophilic predominance and recent neurosurgery.
    3-the CSF protein is markedly elevated, there are 175 lymphocytes and the adenosine deaminase level is elevated.
    4-Petechiae and rash are present on the wrists and ankles that move toward the body. CSF lymphocyte count is mildly elevated.

    pneumococcus is the most common cause of bacterial meningitis. you can't tell the difference between pneumococcus, hemophilus, gram negative meningits, and staphylococcus for sure without culture of the cerebrospinal fluid.

    2-staphylococcus is the most common organism after recent neurosurgery.
    3-TB is suggested by a very high protein, high adnosine deaminase level, and lung lesions.
    4- Rocky mountain spotted fever presents with a salmon colored rash that is vascuilitic in nature and moved toward the body. A tick bite is recalled in 60% of cases. CSF shows a modest elevation in lymphocyte count.

    __________________________________________________ ________

    A patient comes in with fever, headache, photophbia and neck stiffness. The cerebrospinal fluid (CSF) protein level and white cell count are elevated.

    1- an alcoholic, elderly patient who is HIV positive and is on steroids for lymphoma with 2,300 neutrophils in the CSF
    2-Generally a healthy patient with mild lymphocyte elevation.
    3-Adolescent with a petchial rash and terminal complement deficiency. Neureophil count is elevated.
    4-HIV positive pt with mild sx over several weeks. CD4 count of 20. Mild CSF lymphocyte elevation...

    1- Listeria monocytogenes presents with increased neutrohils in the CSF in pts who are immunocompromised, elderly, neonates. steroids, alcoholism, chemotherapy, and leukemia all predispose.
    2-Viral meningitis occurs in healthy pts and is self-limited. there is no past medical hx and the lymphocyte count in the csf is mildly elevated.
    3-Neisseria meningitis occurs more often in adolescents in conditions of crowding such as dormitories or in military recruits. a petechial rash is characteristic. Splenoectomy and terminal complement (C5-C9) deficiency are very strong risk factors. A vaccine against N.meningitis exists.
    4-Cryptococcus in the case of HIV/AIDS with low CD4 counts (<50). cryptococcus gives a modest elevation of lymphocytes in the CSF.

    __________________________________

    A man comes in for evaluation of fever, cough and sputum production.
    1- fever is minimal. the chest x ray is normal.
    2-there is discolored sputum with hemptysis. CXRAY shows an infiltrate in one lobe.
    3-an alcoholic with poor dentition. the sputum is foul smelling.. there is weight loss, with persistent sx over several weeks..
    4-immigrant with weigt loss and a cavitary lesion on cxray

    1-Bronchitis presents with mild fever, cough, sputum, and a normal chest xray.
    2-Pneumococcal pneumonia is the most common cause of community acquired pneumonia. There is discrete infiltrate seen in individual lobes of the lung. Hempptysis is a nonspecific finding. Hemoptysis will not help you answer the Q. anything that makes you cough will give hemoptysis.
    3-Lung abscess is an answer when the sx are chronic over several weeks. the sputum smells bad and there is an increased risk of aspiration such as intoxication, seizures, stroke, or intubation. poor teeth predisposes to higher volume of infected material to aspirate.
    4-TB is most often in immigrants. there is chronic cough, fever, weight loss and night sweats with cavitary lesion.

    _______________________

    a pt comes to the ER with fever and a cough for the last several days. the cxray is abnormal with bilateral interstitial
    1- HIV positive pt with 110 CD4 cells on no medications. The cough is dry and the LDH is elevated.
    2-an 82 year old man with COPD with diarrhea and altered mental status. Sodium is low.
    3-pt is a sheep farmer. Bilateral interstitial infiltrates are present.
    4-Generally a healthy young person, hemolysis is present.

    pneumocystis pneumonia (PCP0 is the answer when the pt is HIV positive with <300 CD4 cells on no prophylactic medications. The LDH is elevated.

    2-Legionella pneumonia is associated with GI and CNS abnormalities in elderly pts with a hx of lung disease. The sodium level is often especially low in pts with legionella pneumonia

    3-Coxiella brunetii causes Q fever. Coxiella is transmitted from animal exposure.
    40 Mycoplasma pneumonia is the answer when the question describes a generally healthy pt with mild sx and interstitial infiltrates. Occasionally there is autoimmune hemolysis from IgM cold agglutinins.
    _____________________________
    32 year old woman comes to the ER with lower abdominal pain and lower abdominal tenderness. cervical motion tenderness
    1-temp is 101F and the white cell count is 16,000, NL 4,500-10,500
    2-pregnancy is positive

    1-pelvic inflammatory disease is the dx when there is lower abdominal pain and tenderness with cervical motion as well as fever and leukocytosis. A pregnancy test should be done to exclude ectopic pregnancy. cervical cultures and DNA probe for gonorrhea and chlamydia should be performed. the most accurate test is a laproscopy. in most cases think about admission for parenteral antibiotics when febrile.
    2-ectopic pregnancy presents with cervical motion tenderness and a positive pregnancy test. A pelvic ultrasound should be performed. if negative than a transvaginal.

    __________________________

    A man comes to the clinic with a genital ulcer and enlarged inguinal adenopathy
    1-ulcer is firm and painless with heaped up and indurated borders.
    2-the ulcer is soft and painful.
    3-large nodes that are tender is the main finding
    4-the ulcer started as vesicles that lost their roof

    1-primary syphilis presents as a genital ulcer with adenopathy. the ulcer is firm but painless. the most accurate test is darkfield exam of a scraping. RPR and VDRL are only 75% sensitive in primary syphilis. rx single IM PCN
    2-chancroid is soft and painful, specialized culture media are necessary to diagnose Hemophilus ducreyi. rx is a single dose azithromycin.
    3- lymphogranuloma venereum presents with matted enlarges lymph nodes. the nodes may develop a draining sinus tract and often tender. dx is with complement fixation testing of a smaple of blood or with aspration of the node.
    rx is with doxycyline for three weeks.
    4-Herpes simplex begins as a vesicular lesions that may ulcerate. if the dx isn't clear, viral culture confirms the dx.
    ___________________________________________
    a pt comes in with dysuria such as urinary frequency, urgency and burning

    1- urethral discharge is present.
    2- there are 50 white cells in the urine, suprapubic tenderness is present.
    3-fifty white cells are present in the urine. temp is 102F an there is flank tenderness.
    4- after rx for pyelonephritis for 7 days, fever, flank tenderness and pyuria persist.


    1-urethritis presents with urethral discharge, although a discharge by itself is sufficient to suggest the diagnosis. A urethral swab for gram stain shows gonorrhea. urine for nucleic acid amplification testing is the standard of care. RX is with a single dose of azithromycin and ceftriaxone. Always treat for chlamydia as well, since rate of coinfection is very high.

    2-cystitis is suggested by dysuria >5 white cells in a urinalysis, and suprapubic pain. Three days of trimerthoprim/sulfamethoxazole or quinolone is the rx
    3-pyelonephritis is diagnoses with dysuria, fever, flank pain and tenderness and white cells in the urine. sonogram or ct of the kidney will sow, possibly hydronephrosis or abcess.
    4-perinephric abscess is diagnosed with persistent sx of pyelonephritis despite rx. imaging of the kidney will show a collection of infected materla. Biopsy is the most accurate diagnostic test.
    ______________________________________
    pt comes wth pruritis of his genital area.
    1-there is also an itchy rash of the web spaces of his fingers, elbows, and axilla. narrow burrows are visible in web space
    2-the itching is limited to hair containing areas of the pubic and axilla. Live organisms are visible near the hair.

    Scabies presents with pruritic lesions of the genitals. There are itchy area in the hands, elbows, wrists in the web spaces. Narrow burrows may be visible where the sarcptes scabiei has dug underneath the skin.
    2-pediculosis or crabs are much larger than scabies and are visible on the skin surface in hair, an pubic region and the axiilla. scabies and pediculosis are best treated with prematherin.

    ______________________________________

    pt comes in with swollen, red, immobile joints

    1- single joint involves in an elderly pts with a hx of arthritis, and there is effusion present.
    2-young pt has multiple joints invlved. there are petechiae, rash, and tenosynovitis present. there is pain on moving the fingers and toes.

    1-Septic athritis from staph or strp presents with involvement of a single joint. Most often the pt has a hx of underlying joint abnormality such as arthritis. the more deformed the joint is the more likely the pt to have septic arthritis. most accurate test is aspiration of the joint for cell count and culture.
    2-disseminated gonorrhea presents with polyarthritis, tenosynovitis and petechia, the most accurate method of establishing a dx is to culture the joint as well as the urethra, cervix pharynx, rectum and blood.

    ____________________________________________


    __________________________________________________ _

    diagnostics

    180pxScabiesburrow 1 - The Medical student Review

    skin scraping mineral oil mount is used to aid in the diagnosis of scabies. mineral oil is applied over suspected lesion to enhance the viewing of a burrow. The skin is scraped and viewed under a microscope. The skin cells remain intact, mite feces are preserved, and the mites remain motile and alive in mineral oil.

    look for pt with itchy pruritic rash scabies1 1 - The Medical student Review
    visible particularly in the web spaces of hands and feet. there may also be a hx of sexual contact and genital or breast involvement as well.

    A KOH wet mount is prepared from the skin scrapings and the slide is heated, breaking up the tissue to reveal any mites hiding under a lump of squamous cells.

    __________________________________

    patchtesting 1 - The Medical student Review

    patch testing is used to determine the allergen responsible for allergic dermatitis after conservative measures like removing the supposed offending agent and a trial of topical therapy fail.

    patch testing isn't useful for irritant contact dermatitis because irritant dermatitis is not an immunologically mediated response.

    _______________________________________
    3890 - The Medical student Review

    wood's light examination used a YV light to diagnose certain skin infections caused by yeast, bacteria, or dermatophytes. it is also used to detect pigment changes in the skin. for instance it detects loss of pigment in fair skin individuals affected with a depigmentation disorder such as vitiligo
    the following organisms fluoresce

    pityrosporum ovale pityrosporum ovale 1 - The Medical student Review will fluoresce yellow
    corynebacterium minutissimum erythrasma 3601 med - The Medical student Review will fluoresce bright red.

    mcanis1 - The Medical student Review microsporum canis will fluoresce blue/green like this you get the pic Microsporum20canis20fig1 - The Medical student Review
    in the scalp but not on glabrous (no hair) skin
    in the U.S tinea capitis is most commonly caused by Trichophyton tonsurans Trichophyton tonsurans 04 1 - The Medical student Review which doesn't fluoresce under wood's light..

    _____________________________________
    gross2 1 - The Medical student Review CNS302 1 - The Medical student Review
    v1c01a 1 - The Medical student Review

    these are CT scans of the head without contrast for the latter two.
    they show intracranial hemorrhage., one with epidural hematoma, the last with a subdural hematoma. the epidural is from an arterial bleed and has higher pressure and more readily pushed in the brain, the blood forms in the shape of a lens. the subdural if a form of venous bleed that forms the shape of a crescent .
    CT is always the best initial test for the brain, especially in trauma when looking for blood. the cr is always best for a stroke in which we are looking for blood.
    in head trauma and intracranial bleeding such as subdural and epidural hematoma, there is no test more accurate than head ct scan.
    _____________________________________

    3901201f1 1 - The Medical student Review

    this is an MRI. it is the most accurate test for MS, strokes, lesions in the brain stem and cerebellum.
    the more accurate test than an MRI for a mass lesion is a brain biopsy.

    ___________________________

    pharm

    54 year old woman comes to the office for eval of sudden overwhelming urge to urinate and pain in her bladder. this happens frequently and results in incontinence. she failed behavior modification for this problem

    the best therapies for overactive bladder or 'urge' incontinence is tolterodine, oxybutynin, trospium, darifenacin or solifenacin.
    These meds are muscarinic receptor antagonists and have significant antichoolinergic effect.
    they are associated with dry mouth, dry eyes and constipation, secondary to their anticholinergic effects.

    ___________________________________

    Varenicline
    ** ask your doc if this is right for you, if you are a smoker
    is a therapy for smoking cessation. it is an adjunct therapy to behavioral modifications, nicotine replacement, and bupropion. It is most useful in people who have signs of nicotine withdrawal. it has greater efficacy than nicotine replacement therapies, such as patches, gum and bupropion.
    Varenicline is a nicotinic ACH receptor partial agonist. It blocked nicotine from binding..

    ______________________________________

    bupropion
    is an antidepressant that can also be used to help with smoking cessation.
    bupropion inhibits the reuptake of norepinephrine and dopamine. it is an aromatic aminoketone that has amphetamine like qualities.
    bupropion can cause HTN, insomnia and seizures but does not have adverse side effects like SSRIs and is contraindicated for ppl with eating disorders because it can suppress appetite. it is also contraindicated for those with a seizure disorder and those using MAO inhibitors.

    ___________________________________

    Thrombin inhibitors

    Argatroban
    Lepirudin
    Hirudin
    Bivaliridun

    direct thrombin inhibitors are used as anticoagulants for pts with clots who have developed heparin induced thrombocytopena and can no longer use haprin. in a pt with heparin drip whose platelet count dropped by half. may also describe a clot or a positive test for plt factor four therefore Argatroban and Lepirudin are the best initial therapy. Bivalirudin and Hirudin can also be used for acute coronary syndromes.
    these meds directly inhibit thrombin
    common adverse effect is bleeding
    they are monitored by checking the activated aPTT.

    __________________________________________

    32 year old man is admitted with his third painful crisis from Homozygous SCD. his crisis has resolved on discharge
    Hydroxyrea is used to decrease the frequency of painful sickle cell crisis. However, hydroxurea does not terminate in an acute crisis.

    hydroxurea increases the level of fetal hemoglobin, preventing subsequent crisis. it also increases the amount of water in the red cells preventing sickling.
    Hydroxurea causes myelosuppression and leukemia with long term use.
    Hydroxyurea is also used to lower cell counts in essential thrombocytopenia and polycythemia vera.
    __________________________________________________ __
    Last edited by جوري; 10-09-2008 at 03:47 AM.
    The Medical student Review

    Text without context is pretext
    If your opponent is of choleric temperament, seek to irritate him 44845203 1 - The Medical student Review


  24. #39
    جوري's Avatar Full Member
    brightness_1
    Soldier Through It!
    star_rate star_rate star_rate star_rate star_rate star_rate star_rate star_rate star_rate star_rate star_rate
    Join Date
    Jul 2006
    Location
    من ارض الكنانة
    Gender
    Female
    Religion
    Islam
    Posts
    27,759
    Threads
    1260
    Rep Power
    258
    Rep Ratio
    89
    Likes Ratio
    23

    Re: The Medical student Review

    pharm heme
    _____________

    Angrelide

    is a platelet reducing agent.
    angarelide is a phospohdiesterase inhibitor

    Anagrelide has a single indication as the rx of thrombocytosis. secondary to myeloproliferative disorders. while anagrelide is used to lower the pt count in essential thromocyteopenia (ET) phone home , chronic myelogenous leukemia (SML) and polycythemia vera. it isn't as effective as hydroxyurea for ET and not as effective as imatinib for CML.

    _____________________________________

    1- a 24 year old woman comes in with epistaxis and petechiae. the plt count is 27,000. she is generally healthy, and her spleen is normal size.
    2-a 45 year old man comes i with melena and intracranial bleeding . his plt count is 8000. his bone marrow shows increased megakaryocytes. He has antibodies present against glycoprotein IIb/IIIa receptor.

    the pt has idiopathic thrombocytopenic purpura (ITP) she is generally a healthy person with only low plt. there is no splenomegaly. treat with steroids. If the thrombocytopenia is repeatedly recurrent aftet stopping steroids, a splenectomy should be performed.
    2-life threatening bleeding with ITP is treated with IVIG or rhogam in combination with steroids. The fastest way to bring up the plt count is with IVIG. this rx is faster than plt transfusion in which the plts will be consumes.. Plasmapheresis isn't helpful.

    __________________________________________

    plasmapheresis
    is the best initial therapy for the following

    Guillain Barre syndrome. equal in efficacy as IVIG
    Goodpasture's syndrome, in combo with steroids
    Myasthenia gravis crisis in which there is an overwhelming svere muscle weakness. IVIG can be used in this case as well
    Thrombotic thrombocytopenic purpura TTP
    waldenstrom's macroglobulinemia. used to dec hyper viscosity from the high levels of IgM

    _______________________
    22 year old man comes to the office for increased bleeding after using asa. he has some petechiae on his legs, bleeding time is prolonged. the ristocetin cofactor assay test is abnormal. He is about to undergo a tooth extraction

    Desmopressin DDAVP is the best initial therapy for von willebrand's dz. desmopressin is an artificial version of ADH or vasopressin.
    Desmopressin works by causing the release of subendothelial stores of von willebrand's factor VIII

    Desmopressin can lead to HTN, facial flushing, and headache. Desmopressin can also be used for mild hemophilia and with central diabetes insipidus.

    _____________________________________

    a woman with factor V leiden mutation develops a PE. she is hemodynamically stable

    2- man with rate conteolled Afib is stable at home, he is 68 with dilated left atrium on echo.

    1- PE are best treated with heparin; IV-unfractionated heparin and low molecular weight heparin have equal efficacy. warfarin should also be started and heparin continued until warfarin is in therapeutic range.
    2- warfarin is continued for 6 months. the duration of therapy doesn't change because of the presence of factor V mutation with the first episode of clot.
    IV heparin is monitored with the activated aPTT. LMW heparin is occasionally monitored with factor xa levels

    case II

    chronic afib is anticoagulated with warfarin to an internatioalized normalized ratio INR of 2-3, there is no need to start heparin before starting warfarin.
    warfarin is continued permanently.
    warfarin is monitored with INR.

    ______________________________________

    45 year old male GI has afib. he has a heart rate of 60-80 bpm, no htn, dm, previous strokes.. the echo is normal.

    afib can be rx with asa alone. this is a long afib asa alone for alone afib. the pt is without accompanying risks, such as dm, htn, previous stokes. and has a normal echo. low risk pts would have al the bleeding risks of the drug without the therapeutic benefit.

    ____________________________

    A pregnant woman develops PE in her 2nd trimester

    thromboembolic dz during pregnancy is treated with heparin throughout the pregnancy. because of the possibility of teratogenicity with warfarin. LMW heparin can be administered safely by subcutaneous injections.
    heparin works by potentiating the effect of antithrombin on the clotting cascade.

    Besides bleeding, heparin is associated with thrombocytopena, long term use can be associated with osteoporosis

    _________________

    43 year old man comes to the hosp. w back, thigh, and chest pain and weakness, he has a temp of 102F. he has enamia and an elevated bilirubin, LDH, and retic count. fluids, pain meds and o2 have been started,

    this pt has sickle cell dz with an acute painful crisis. He is febrile. the most important med for is at this time is antibiotics. Ceftriaxone, levofloxacin, gatifloxacin or moxifloxacin are all appropriate to cover encapsulated organisms such as strep pneumo, klebsiella, salmonella, or hemophilus.

    fever in sickle cell pt is life threatening, because the pt is functionally asplenic. do not wait for results. pt will die of sepsis before the results are known.
    admin of antibiotics for fever lowers mortality as well hydroxyurea to prevent crisis.

    ___________________________

    Anti-Ach-R antibodies
    Myasthenia gravis is characterized by anti ACH receptor antibodies. that are made when autoimmune dz causes antibodies that bind to postsynaptic ach receptors: there is no neuromuscular transmission if there is no receptor on the muscle because it has been eaten away by an antibody.

    Anti-Ach-R antibodies are the best test to dx Myasthenia gravis. they are more accurate than tensilon or edrophonium testing.
    single fiber electromyography EMG is the most sensitive diagnostic test for MG.

    ____________________________

    0001 0002 1164809208 0001 1 - The Medical student Review

    carotid doppler is a sonographic eval of the carotid arteries that allows a precise quantification of the degree of stenosis of these vessels. it is used to see whether a pt needs a carotid endarterectomy or angioplasty.
    carotid doppler is used on pts who present with signs of ischemia in the anterior circulation of the brain, such as amaurosis fugax of focal extremity weakness. it is the initial test to determine the etiology of embolic strokes or TIAs

    the most accurate test for carotid arteries is the angiogram.

    _____________________________________
    cervical spine x ray
    82188704 6b1dbba387 1 - The Medical student Review
    for someone diving and landing on his head or having somethin heavy land on his head. gives increased axial loading.

    _________________
    ct myelogram
    CTAxial Tuto L5Bone 1 - The Medical student Review
    is to detect compression of the spinal cord from mass lesions, such as cancer and occasionally from hematoma or an infection such as epidural abscess.
    a spinal needle is introduced into the subarachnoid space. the pt is hung nearly upside down in order to allow contrast material into the subarachnoid space to move and detect the lesion at the place it stops flowing. The MRI has essentially replaced the CT myelogram. though it may has an advantage in detecting leptomeningeal cancer.

    MRI will always be the better answer, if you suspect cord compression.
    if MRI isn't possible b/c pt can't lie still or has a metal pacemaker and the magnet of the MRI would be dangerous to such a patient.

    ____________________________

    Edrophonium tensilon test

    is a very sensitive test for MG. Edrophonium chloride is an ACH esterase inhibitor with rapid onset and short duration of axn. it prolongs the presence of ach in the neuromuscular junction and results in an immediate increae in muscle strength
    elecromypgraphy is the single most accurate test to confirm the dx of MG
    thymomas are associated with the dx of MG 10% of the time.

    ____________________

    EEG
    website EEG 764486 - The Medical student Review
    is the measurement of brain wave patterns by putting electrodes on the scalp.
    done in pt with new onset of seizure, also elpful in pts with sudden onset syncope if there is no cardiac etiology identified.

    EEG maybe contributory in the dx of Creutzfeldt-Jakob dz. but is isn't as good as a brain biopsy. EEG doesn't necessary determine brain death if the brains tem reflexes are clearly absent.

    Sleep deprivation EEG has the greatest sensitivity.. this is the answer is the pt has been seizure free for 2-3 yrs and you want to stop anti epileptic meds.

    ____________________

    EMG

    emg - The Medical student Review
    is a test of muscular strength and electrical activity. EMG's are used to dx neuromuscular dz. to distinguish between neural and muscular disorders.
    Needle is places in the muscle. The electrical activity during rest and activity is compared. An abnormal pattern or amplitude tells there is dz.
    an EMG in the case should make you think of MG or Guillain Barre syndrome and muscular dustrophy.
    single fiber EMG is the single most accurate test for MG. combined EMG and nerve conduction velocity testing is the single most accurate test for GBS. EMG combined with muscle biopsy are the most accurate tests for polymyositis.
    ____________________________

    Lumbar puncture

    pain lumbar puncture 1 - The Medical student Review
    is insertion of a spinal needle into the subarachnoid space to aspirate CSF fluid.
    for the following
    suspected meningeal infxn
    pt with worst headache of her life, whom CT is negative to exclude subarachnoid hemorrhage.
    new onset seizures with fever
    any infant less than 6 weeks old with fever if other studies are negative,
    LP's are contraindicated for mass lesion. do use a head CT before LP when there are focal findings, papilledema, or a new seizure, or when pt is confused as to make the neurological exam inaccurate.

    ____________________________________

    oligoclonal bands

    NU9 - The Medical student Review

    detection of elevated bands of igG (oligoclonal bands) in the csf is possible in a pt presenting with signs and sx of MS. the presence of these bands isn't specific.

    look for a young pt with weakness, numbness, unsteady gait, spastic paraparesis, diplopia, or sphincter disturbances (urinary urgency_ exxamining the CSF for oligoclonal bands is recommended only when MRI is nonconfirmatory but your clinical suspicion for MS remains high.

    MRI of the brain is the most accurate test to dx MS wit a sensitivity of 90-95%

    >>>>>>>>>>>>>>>>>>>>>.
    The Medical student Review

    Text without context is pretext
    If your opponent is of choleric temperament, seek to irritate him 44845203 1 - The Medical student Review


  25. Report bad ads?
  26. #40
    جوري's Avatar Full Member
    brightness_1
    Soldier Through It!
    star_rate star_rate star_rate star_rate star_rate star_rate star_rate star_rate star_rate star_rate star_rate
    Join Date
    Jul 2006
    Location
    من ارض الكنانة
    Gender
    Female
    Religion
    Islam
    Posts
    27,759
    Threads
    1260
    Rep Power
    258
    Rep Ratio
    89
    Likes Ratio
    23

    Re: The Medical student Review

    Infectious disease
    ___________________________

    a pt comes to the hospital with fever and a murmur

    1- 67 year old woman with four months of fever and fatigue. she has an HX of MR
    2-a 27 year old injection drug user, the murmur is best at the lower left sternal border.
    3- A man whose aortic valve was replaced three weeks ago
    4-a pt who was recently diagnosed with diverticulitis and colon cancer.

    Virdians group streptococci is the most important organism in subacute bacterial endocarditis. They occur most often in those with underlying valvular disease.
    2-staphylococcus aureus is the most common cause of endocarditis in the injection drug user. this is often MRSA as well Oxacillin resistant. injection drug users often have involvement of the right side of the heart such as the tricuspid.
    3-Staphylococcus epidermidis and other coagulase negative staphylococci are the most common cause of endocarditis when a heart valve has been replaced. This is presumably from seeding of the valve during surgery.

    4-Staphylococcus bovis is most often associated with endocarditis in those with evidence of colonic pathology.

    _____________________________________________

    an HIV positive man with 25 CD4 cells comes in for blurry vision x a few days. He is not on any HIV meds

    CMV retinitis occurs exclusively in pts with CD4 counts under 100. CMV presents with blurry vision.
    2-Dilated ophthalmologic examination is the best initial method of diagnosing CMV retinitis. It is basically diagnosed on how it looks. CMV antibody testing in the blood has no value. It is a clinical dx based on direct retinal visualization.

    Ganciclovir, foscarnet or valganciclovir is the standard of care in rx.
    __________________________________________________ __

    pt is admitted to the ICU because of severe metabolic acidosis. The serum bicarbonate is low at 14. the pt is disoriented and can't offer an adequate hx. no records are available.

    1- fever, hypotension, tachycardia, and an elevated white count
    2- hyperglycemia and hyperkalemia
    3-oxalate crystals in the urine with a low serum calcium
    4-elevated CR.
    5-Normal anion gap

    fever, hypotension, leukocytosis, and tachycardia imply the presence of sepsis as a cause of metabolic acidosis. The first step in evaluation of an metabolic acidosis is the eval of the anion gap. An anion gap (na+ minus Cl- and Hco3-) that is >12 is consistent with lactic acidosis, salicylate OD, Methanol, uremia, diabetic ketoacidosis, and ethylene glycol OD.
    2-Diabetic ketoacidosis (DKA) gives hyperglycemia and hyperkalemia, although the total body levels of potassium are depleted.
    3-Ethylene Glycol OD results in oxalate crystals in the urine. The formation of calcium oxalate crystals lowers the calcium level. look for the term envelop shapes crystals
    4-Renal failure causes metabolic acidosis because of the kidneys inability to excrete acid.
    5-normal anion gap implies either RTA or diarrhea, the urine anion gap is positive. With diarrhea, the urine anion gap is strongly negative. The lower the urone anion gap number, the greater the kidney's ability to excrete acid.

    ___________________________________

    A man is admitted to the hospital with renal failure developing over a few days
    his cr has risen from 0.8mg/dl to 2.5 mg/dl. His bun has risen even more, going from 15 to 54. his serum bicarbonate is slightly low. the urine sodium is low and the urine osmolality is high..

    Blood pressure 92/56 and pulse is 124.
    2-serum albumin is 2.2 and the PT is elevated. there is splenomegaly
    3-Ha has an ejection fraction of 24% with edema. a diuretic was recently started.
    4-a bruit is present at the flanks and he has just started an ACE inhibitor.


    will take care of the rest of this tomorrow insha'Allah.. am feeling a bit tired today..

    The Medical student Review

    Text without context is pretext
    If your opponent is of choleric temperament, seek to irritate him 44845203 1 - The Medical student Review



  27. Hide
Page 2 of 7 First 1 2 3 4 ... Last
Hey there! The Medical student Review Looks like you're enjoying the discussion, but you're not signed up for an account.

When you create an account, we remember exactly what you've read, so you always come right back where you left off. You also get notifications, here and via email, whenever new posts are made. And you can like posts and share your thoughts. The Medical student Review
Sign Up

Similar Threads

  1. Replies: 2
    Last Post: 05-26-2010, 02:35 PM
  2. Book Review
    By Beardo in forum Creative Writing & Art
    Replies: 2
    Last Post: 11-13-2009, 03:51 PM
  3. Replies: 1
    Last Post: 08-28-2007, 11:08 PM
  4. Replies: 0
    Last Post: 04-21-2007, 01:10 PM
  5. book review
    By partysoverkids in forum General
    Replies: 1
    Last Post: 02-19-2006, 10:50 PM

Posting Permissions

  • You may not post new threads
  • You may not post replies
  • You may not post attachments
  • You may not edit your posts
  •  
create